Sei sulla pagina 1di 53

Alberta High School Mathematics Competition

Solution to the First Round, 2006.


1. The value of 24 48 816 is 24+82+163 = 268. The answer is (c).
2. There are 4 such rectangles, namely, 1 2006, 2 1003, 17 118 and 34 59. The answer
is (b).
3. Since 1004 = 100000000, (100,1) is a desired pair. In fact, m can be any positive integer up to
100, and there is a unique positive integer n such that m4 + n = 100000001 for this particular
m. Hence the total number of pairs is 100. The answer is (a).
4. A and C together travel as many north-south blocks as B and D together. A and C together
also travel as many east-west blocks as B and D together. Hence A and C together travel
as many blocks as B and D together. It follows that the number of blocks D travels is
10 + 50 20 = 40. The answer is (d).
5. We are counting the number of ways of making 9 from 1, 2, 3, 4 and 5, using each number at
most once. There are 3 ways, namely 1+3+5, 2+3+4 and 4+5. The answer is (d).
6. Draw a circle with centre A and radius 2. Draw another circle with centre B and radius 3.
The lines we seek are the common tangents of these two circles, of which there are 3. The
answer is (d).
7. We can steal the election by winning as little as 16 votes. We can win 2 provinces, 2 cities
within each of these 2 provinces, 2 wards within each of these 4 cities, and 2 electors within
each of these 8 wards. Hence (81 16) + 1 = 66 votes are required to guarantee winning the
election. The answer is (c).
8. Let A and B be the respective midpoints of two opposite edges of length 9. Let C and D be
the respective centres of the two 7 8 faces. (See the daigram below.) Then AB and CD
intersect at the centre O of the box. Place a coin of diameter 9 on the plane determined by
AB and CD with centre O.Its circumference will pass through C and D, but A and B are
not covered up since AB = 72 + 82 > 9. Rotate the coin about AB so that it just comes off
C and D. We can then expand the coin slightly and still have it fit inside the box. The answer
is (e). We do not know the actual maximum value. That is why the question is phrased in its
current form.
.
.......................
.............
.....
..............
.....
.............
.....
.............
.
.
.
.
.
.....
.
.
.
.
.
.
.
.......
.
.
.
.
.
.
.
.
.
.
.
.
.
..... ......................................................................................................................................................
.
.
.
.
.
.
..
.
.
........
.
.
.....
.
.
.
.
....
.
.
.
...
.
....
.
...
.
.
.
.
.
.
.
....
....
....
..
..
.
.
.
.
...
..
...
...
..
.
...
.
.
...
...
..
...
.
.
.
....
....
..
.
.
.
.
...
..
.
.
...
...
....
...
...
..
...
...
....
...
..
..
...
...
..
.
....................................................................................................................................................
....
...
....
....
...
.
...
..
.
.
...
..
..
....
....
...
..
..
.
.
...
.
...
.
..
..
..
..
.
.
...
...
.
.
..
.
.
.
.
...
...
...
.
.
...
....
...
...
.
..
....
.
.
.
.
...
...
.
....
...
.
.
...
.
.
...
.....
...
.
.
.......
.
....
...
..
.
..
...............................................................................................................................................................................

9. Solving for b, we have b(1 a) = 1 a. Since a is not an integer, 1 a 6= 0 and can be


cancelled. Hence b = 1, and is a positive integer. The answer is (b).
10. Let xa = yb = zc = r. Then xyz
= r3 , x = ra, y = rb and z = rc. It follows that x+y = r(a+b),
abc
= y+z
= z+x
= r. The given expression is
y + z = r(b + c) and z + x = r(c + a), so that x+y
a+b
b+c
c+a
3 1 3
equal to r ( r ) = 1. The answer is (c).

11. We have 9 3 11 2 = ( 3 + k 2)3 = 3 3 + 9k 2 + 6k 2 3 + 2k 3 2. From 9 = 3 + 6k 2 ,


we have k = 1. From 11 = 9k + 2k 3 , k < 0. Hence k = 1. The answer is (b).
12. We change the base-ten number 50 to base-two and obtain the number 110010, representing
25 + 24 + 22 . We now interpret this as a base-five number, representing 55 + 54 + 52 . Changing
this number to base-ten, we obtain 3755. The answer is (b).

13. Since 32 and 23 are roots, we must also have 32 and 23 as roots in order to have rational
coefficients. Thus the degree of the polynomial cannot be less than 4, but 4 is sufficient. Of
these polynomials, the one with 1 as the leading coefficient is
!
!
!
! 


2
3
35
1
2
2
3
3
2
2
x
x+
x
x+
= x
x
= x4 x2 + .
3
3
2
2
9
4
36
6
Since we want integral coefficients, we clear the denominators to obtain 36x4 35x2 + 6. Since
the coefficients are relatively prime (though not pairwise relatively prime) and the smallest
absolute value among the coefficients is 6, the smallest positive coefficient is also 6. The
answer is (b).
= 25 or
14. We have a + bc = 11 and b + ac = 14. Adding the two equations yields (a + b) c+1
c
(a + b)(c + 1) = 25c. Since c + 1 and c are relatively prime, c + 1 must divide 25. Hence c = 4
or c = 24. If c = 24, then a + b = 25 but at least one of ac and bc is not an integer. Hence
c = 4, a + b = 20 and a+b
= 5. The answer is (b).
c
15. Let the base of
the equilateral triangle ABC. Let D be the midpoint of BC.
the pyramid be
2
2
Then AD = AB BD = 23 . Let G be the centre of ABC. Then G lies on AD and

3
=
. Let V be the fourth vertex of the pyramid and O be the centre of the sphere
AG = 2AD
3
3
passing through all four vertices. Then O lies on V G and let r = V Oq= OA be the radius of

the sphere. Since V G V O = OG = OA2 AG2 , we have 1 r = r2 13 . Squaring both


sides, we have 1 2r + r2 = r2 13 . It follows that r = 23 . The answer is (e).
16. Note that p(2) < p(1) > p(0) < p(1) > p(2). Hence the graph of this fourth degree
polynomial opens down. Let p(x) = x4 + ax3 + bx2 + cx + d. Then d = p(0) = 2. From
p(2) = 2, we have 168a+4b2c+2 = 2. Hence 4b16 = (8a+2c). This is only possible
if both sides are equal to 0, so that b = 4. Similarly, from p(1) = 5, we have 25a + 5c = 0.
2
Combining with 8a + 2c = 0, we have a = c = 0, so that p(x) = x4 + 4x2 + 2 = 6 (x
2)2 .

It follows that the maximum value of p(x) is 6, occurring when x2 2 = 0 or x = 2. The


answer is (b).

Alberta High School Mathematics Competition


Solution to Part I 2007
1. The number 111111 is divisible by 1001. Now 1001 = 6166+5. Hence the desired remainder
is the same when we divide 11111 by 1001, which is 100. The answer is (d).
2. Tell the cats to put their front legs up. Now there are still 34 heads, but only 34 2 = 68 legs
on the ground. Hence 80 68 = 12 legs are up in the air, and each cat puts up 2 of them. It
follows that the number of cats is 12 2 = 6. The answer is (a).
3. Since AB 1 and BC 2, the area of triangle ABC is at most 2. This maximum value
can
be attained when AB = 1 and BC = 2 and are perpendicular to each other. Now CA = 5,
and we indeed have 2 CA 3. The answer is (a).
4. The middle symbol must be an A. The first five symbols consist of two As and three Bs, and
they can be arranged in all possible ways. The last five symbols, consisting also of two As
and three Bs, must be in reverse order with respect to the first five symbols. In the first five
symbols, we only have to find
 the number of ways of choosing two of the five positions for the
5
two As. This is given by 2 = 10. The answer is (c). An exhaustive analysis also works.
5. The integers from 11 to 30 include six primes, namely 11, 13, 17, 19, 23 and 29. In the ten odd
numbers among any twenty consecutive integers each at least 7, at least three are multiples
of 3 and exactly two are multiples of 5, but at most one can be a multiple of 15. Hence the
maximum is indeed six. The answer is (c).
6. We have 2x+2y = 5+y 5, so that the minimum value of x+y is 52 , attained at (x, y) = ( 52 , 0).
Also, x + y = 5 x 5, so that the maximum value of x + y is 5, attained at (x, y) = (0, 5).
The answer is (d).
7. If we take all the even numbers, clearly no two will differ by 3 or 5. Hence we can take at least
500 numbers. Now partition the integers from 1 to 1000 into blocks of 10. From each of the
following five pairs, we can take at most one number: (10n + 1, 10n + 4), (10n + 2, 10n + 5),
(10n + 3, 10n + 8), (10 + 6, 10n + 9) and (10n + 7, 10n + 10). Hence we can take no more than
500 numbers. The answer is (d).
8. Suppose there exists such a polynomial p. Since an bn is divisible by a b for all positive
integers a, b and n with a 6= b, 13 9 must divide p(13) p(9). However, 4 does not divide
7, and we have a contradiction. The answer is (a). A parity argument also works.
9. The centre O of the circle lies on the axis of symmetry of ABCD. Let y be its height above
BC. Then OB 2 = y 2 + 32 while OA2 = (8 y)2 + 12 . Equating these two values yields y = 72 .
Hence the radius is

( 72 )2 + 32 =

85
.
2

The answer is (c).


A
.................... .. D
.............. ............. .. ....................
........ . .. ..
.

.....
.
..
..
....
.. ... ...
.....
...
....
.. ... ...
....
...
...
...
.. .. ..
...
..
...
.. ... ....
.
...
.
.
.
.. ...
..
..
....
...
.
..
.. ...
..
.
.
.
..
..
.. ..
..
...
...
.
.
.
..
.. ...
..
..
...
...
...
..
...
....
...
.
.
....
..
...
...
...
...
..
.
.
.
..
....
.
.
..
..
... ...
.
.
...
.
.
.
..
.
..
.. ...
.
.
..
.
...
.
.
.
..
.
..
..
.
.
.
.
.
.
.
.
. ...
..
..
..
...
....
..
... ...
..
...
...
.. ...
..
...
... ... .....
.. .....
...
.... ......
.
.
.
.
.
.
....................................................................................
.....
.
.......
.....
...........
......
....................................

10. Let the positive integral roots be r s t. Then x3 10x2 + ax + b = (x r)(x s)(x t).
Expansion yields x3 (r + s + t)x2 + (st + tr + rs)x rst. Hence r + s + t = 10. The possible
partitions are (1,1,8), (1,2,7), (1,3,6), (1,4,5), (2,2,6), (2,3,5), (2,4,4) and (3,3,4). The answer
is (b).
4
= 8. This may be rewriten as
11. Multiplying one equation by the other, we have xy + 4 + xy
2
2
0 = (xy) 4xy + 4 = (xy 2) . Hence xy = 2. The answer is (c).

= 2. Since s + c = 1, 2 c = 2c
12. Let s = sin2 and c = cos2 . We have sec2 + tan2 = 1+s
c
= 5. The answer is (d).
so that c = 23 . It follows that s = 13 . Now csc2+cot2 = 1+c
s
13. Let E be the point on AD such that BE is perpendicular to AD. Complete the rectangle
BEDF . Now AB = BC, 6 AEB = 90 = 6 CF B and 6 ABE = 90 6 CBE = 6 CBF . Hence
ABE and CBF are congruent triangles and they have equal area. It follows that BEDF is
a square, and its area is also 16. Hence BE = 4. The answer is (c).
B

.............................................
..........
.......
.......
....
....
....
.
.
.
.
....
....
....
...
...
.
.
...
..
.
.
...
..
.
..
...
..
..
...
...
...
..
..
..
....
...
...
..
..
...
...
..
.
..
.
.
.
..
..
..
..
..
...
..
...
..
..
...
.
...
....
....
....
....
.....
....
.......
.....
.
.
.
.
.
.
.........
.
...............................................

QQ

Q
Q

QC

14. The number of incorrect answers for each of questions 1 and 6 is 0 or 5. The number of
incorrect answers for each of questions 2 and 5 is 1 or 4. The number of incorrect answers
for each of questions 3 and 4 is 2 or 3. A total of 8 incorrect answers can only be made up
from 0+1+3+3+1+0. However, we would have an equal number of incorrect answers of T
and incorrect answers of F. Hence the total must be 9, and it can be made up from either
0+1+2+2+4+0 or 0+4+2+2+1+0. However, the latter yields more incorrect answers of F
than incorrect answers of T. It follows that the correct answers for the six questions are F, F,
F, T, F and T respectively. Only student #4 has both an incorrect answer of T (for question
5) and an incorrect answer of F (for question 4). The answer is (d).
15. We have 1099 59 n (10100 1) 59 . Since n is an integer, 5 1098 < n < 5 1099 . However,
we must eliminate those values of n where 51098 < n < 1099 . Thus the number of acceptable
values of n is about 4.5 1099 . Since 1099 n 10100 1, the desired probability is very
close to 12 . The answer is (c).
16. We can rewrite the inequality as (k 3)(x4 + y 4 + z 4 ) + (y 2 z 2)2 + (z 2 x2)2 + (x2 y 2)2 0,
from which it is clear that k 3. The answer is (c).

Alberta High School Mathematics Competition


Solution to Part I 2008
1. In order for m to be an integer, n must be a multiple of 5. Since there are 19 integer values
of n within range, the number of corresponding values of m that are integers is also 19. The
answer is (b).
2. When a+2b+3c is divided by 13, the remainder will be the same as when 9+27+310 = 53
is divided by 13. The latter is clearly 1. The answer is (a).
3. The total length of all but one side in a polygon with positive area must be greater than the
length of that side. To minimize the perimeter, choose that side to be the longest. Hence the
perimeter is greater than 20. Since all side lengths are integers, the minimum is at least 21.
This minimum value may be achieved with a triangle of side lengths 5, 6 and 10. The answer
is (d).
4. Note that f (x) = x3 and g(x) = x6 satisfy the hypothesis. In any case, we have f ( 12 ) = g(1) =
2f (1) = 2g(2) = 6. The answer is (e).
5. From the first condition, the desired remainder is one of 6, 13, 20 and 27. From the second
condition, the desired remainder is one of 1, 5, 9, 13, 17, 21 and 25. The only common value
is 13. The answer is (d).
6. Consider a horizontal cross-section of the room as a clock, and let the ray start at the 12
oclock position. It must travel directly at another point on the clock in order to return to
its starting position after 11 bounces. However, some of these 11 paths take it back to the
starting point before 11 bounces. The initial destination must be a point which is relatively
prime to 12, namely, 1, 5, 7 or 11. The answer is (a).
7. We have ||||x 1| 2| 3| 4| = 5. Now |||x 1| 2| 3| = 4 5. Since the left side can never
be equal to 1, we must have |||x 1| 2| 3| = 9. Similarly, ||x 1| 2 = 12, |x 1| = 14
and x = 1 14 = 13 or 15. The answer is (c).
8. Since 2008 = 2 2 2 251, the box must be one of 2 4 251, 2 2 502, 1 8 251,
1 4 502, 1 2 1004 and 1 1 2008, with respective surface area 3028, 4024, 4534,
5028, 6028 and 8034. The answer is (a).
9. We have b a = 11 and b2 a2 = 1001, so that b + a = 91. Hence 2b = 102 and b = 51. The
answer is (b).
10. Let Lindas walking speed be v kilometres per hour. Then the length of the trains is given by
2
V . The answer is (a).
t1(V v) = t2 (V + v). It follows that v = tt11 t
+t2
11. The smallest value of a leads to the smallest value of b, and of c. The smallest value of a is
10, a 1 followed by 2 1 0s. The smallest value of b is 109 , a 1 followed by 10 1 0s. The
9
smallest value of c is 1010 1 , a 1 followed by 109 1 0s. The answer is (b).

12. If we take both senior boys, we must complement them with both junior girls. If we take
neither senior boy, we must take both senior girls and both junior boys. If we take only one
senior boy, then we must take one senior girl, one junior girl and one junior boy. In each
category, there are two choices. Hence the number of different teams is 1 + 1 + 24 = 18. The
answer is (e).
13. The height of the circle is 2r where r is its radius. From r2 = 1, we have 2r = 2 . Since

2 < < 4, this is greater than 24 = 1, the height of the square, and less than 22 = 2, the

where 2s
height of the right isosceles
The height
of
the equilateral triangle
is 3s

triangle.
4
4
4
2
is its side length. From 3s = 1, we have 3s = 3. Since 3 < 4, 3 < 4 = 2. The
answer is (b).
14. Let a, b and c be the roots. Then a + b + c = 0 and bc + ca + ab = 3. It follows that
a2 + b2 + c2 = (a + b + c)2 2(bc + ca + ab) = 6. Since x3 = 3x 1, we have
x5 = x2(3x 1) = 3(3x 1) x2 = 3 + 9x x2 .
Hence a5 + b5 + c5 = 9 + 9(a + b + c) (a2 + b2 + c2 ) = 15. The answer is (a).
15. In moving from city to city, we either pick up an extra prime or discard one. The numbers of
prime divisors of 99 = 3 3 11 and 100 = 2 2 5 5 are 3 and 4 respectively, and there
are no common divisors. To change from one lot to the other, we need to make 7 moves. The
answer is (d).
16. We can guarantee that at least one ticket wins by buying the tickets (1,2), (1,3), (2,3) and
(4,5). If neither 4 nor 5 is drawn, the last ticket wins. If at least one of them is drawn, then at
most one of 1, 2 and 3 is drawn. Then one of the first three tickets wins. If we only buy three
tickets, we have to choose 6 numbers. By the Pigeonhole Principle, one of the numbers 1, 2,
3, 4 and 5 will appear on at least two of our tickets. If it appears on all three, none of them
wins if this number is drawn. Suppose it appears only on the first two tickets. If this number
is drawn along with one of the numbers on our third ticket, we will not have a winning ticket.
It follows that the minimum number of tickets to guarantee that at least one of them wins is
4. The answer is (b).

Alberta High School Mathematics Competition


Solution to Part I 2009
1. If 2x = 3y , then 4x = (2x )2 = (3y )2 = 9y . The answer is (d).
2. The number of bones Caroline bought is a multiple of 8 but 2 less than a multiple of 7. The
answer is (d).
3. The calculation is (n+1)+(n+2)++(n+100)
n = 100n+(1+2++100)
n = 1+2++100
, with n = 0
100
100
100
for Ace, n = 1000 for Bea and n = 1000000 for Cec. The answer is (e).
4. Almost the entire gymnasium floor may be divided into 2 3 non overlapping rectangles each
with exactly one non-blank square at the lower left corner. The answer is (c).
5. Observe that x2 y 2 = (x y)(x + y) is the product of two integers of same parity. Hence
x2 y 2 is either odd or divisible by 4. Thus a number which is neither odd nor divisible by
4 cannot be expressed as a difference of two squares. On the other hand, if n is odd, then
n = 2k + 1 = (k + 1)2 k 2 . If n is divisible by 4, then n = 4k = (k + 1)2 (k 1)2 . Between
1 and 2009 inclusive, there are 1005 numbers that are odd and 502 that are divisible by 4.
The answer is (d).
6. We can choose any six of the nine non-zero digits. The number of choices is
choice gives rise to a unique number. The answer is (b).

 
9
6

= 84. Each

7. Assume that the first A appears before the first B, and the first B before the first C. Then we
must start with AB and continue with A or C. If we continue with A, the last three letters
must be CBC. If we start with ABC, we must continue with A or B. In either case, either of
the last two letters can appear before the other. So the total is 1 + 2 2 = 5. Relaxing the
order of appearance, the total becomes 5 3! = 30. The answer is (a).
8. Since 10n1 < 22009 < 10n and 10m1 < 52009 < 10m , we have 10m+n2 < 2200952009 < 10m+n .
It follows that 10m+n1 = 2200952009 = 102009. Hence m + n 1 = 2009. The answer is (d).
9. Let ABC be the triangle and DREP F Q be the hexagon, as shown in the diagram below.
Triangles AP E, AP F, ERD and F QD are all congruent to one another.
Hence DREP F Q
has the same area as the parallelogram AF DE, which is one half of 2 3. The answer is (c).
A
..........
... ... ...
.. ... ...
... ... ...
... .... ....
.
...
...
.
...
...
...
..
...
..
.....
...
..
.
.
.
.
.
.
..
.
.... .........
..
.
.
.
.
.
.
...... ....
.. ...........
.
.
.
...... ...
...........
...... ..
.
........
.....
.....
........
.........
.. .. ..
.. .. ...
.. .... ....
... .... ....
.
.
.
.
.. .... ....
.. ... ....
.
.
...
.
...
..
...
.....
...
...
..
....
..
..
..
...
..
..
..
..
.
.
...
.
.
.
.
.
...
...
.
.
..
...
.
.
.
.
.
...
.
.
..
...
.
........
.
.
.
.
.
...
......
....
.
..
.
..
.
.
.
.
.
.
.
.
......
...
...
. .........
..
.
.
.
.
...
.
.
...... .. .. ......
.
.
.
.
.
..
.
.
.
.
.
...
..
.....................................................................................................................................................................

10. By the Triangle Inequality, the third side must be from 7 to 15. Now 92 < 112 52 < 102 , so
that (5,7,11), (5,8,11) and (5,9,11) are obtuse triangles. Also, 122 < 112 + 52 < 132 , so that
(5,11,13), (5,11,14) and (5,11,15) are obtuse triangles. The other three are not. The answer
is (c).

11. Since Q(9) = 2009, x 9 is a factor of Q(x) 2009. Since Q(x) has integral coefficients, p 9
divides Q(p) 2009 = 392 2009. Since this number is odd, p 9 must be odd. Since p is
prime, we must have p = 2. Now 392 2009 = 1617 = 231 7, so that Q(x) does exist.
For instance, we may have Q(x) = 231x 70. The answer is (a).
12. Let 5 centimetres be the height of the parallelogram. Its base is a segment intercepted by the
other pair of parallel lines 8 centimetres apart. Hence its length is at least 8 centimetres and
can be arbitrarily large. The answer is (b).

13. q
With one square root sign, n < 10 is equivalent to n < 100. With two square root signs,

n + n < 10 is equivalent to n + n < 100, which is in turn equivalent to n < 91. With
r
q

three square root signs and n < 91, we have n + n + n < n + 10 10. With more
square root signs, the same inequality will hold. There are 90 positive integers which satisfy
n < 91. The answer is (b).
14. Let x be the distance from the centre of the circle to the bottom edge of the larger square.
2
The square of the radius of the circle isgiven by 6
+ x2 = 22 + (4 + 12 x)2 . This yields
x = 7 so that the radius of the circle is 62 + 72 = 85. The answer is (d).
......................................................................................................
.................
... .............
.........
....
....
........
...
........
.
.
.
......
.
...
......
.
...
..
.....
.....
...
... ..
.
.....
.....
...
.
.
.
.
.....
....
.
.
.
..
..
.. ..
....
.
.
.
.
.
.
..
....
...
..
.. ...
.
.
.
.
.
....
....
.
. ..
..
.
.
.
...
.
.
.
.
.
. ..
..
.
.
.
...
.
.
.
.
.
...
..
..
.. ....
...
.
.
.
..............................................................................................................................................................................................................
.
.
..
..
...
.
.
.
.
.
.
...
.
.
.
.
.. ..
.
.
..
.
.
..
.
.
.
.. .
.
.
..
.....
.. ...
....
..
..
.
.
.
..
.
.
.
.
..
.
..
.
.. ...
.
.
..
.
.
.
... ..
...
.
...
.
...
.
.
.. ..
..
....
..
..
.. ..
.
...
..
... ...
...
..
.
..
.. ...
....
...
...
..
....
.
..
....
.
..
......
..
....
...
.
...
.
...
..
.
.
....
...
...
.
...
.
.
.
...
.
.
.
..
...
... ...
..
..
.
...
.
.
.
...
. ...
.
.
.
.
...
.
.
.
.
.
..
.
.
..
.
..
.
.
.
.
.
.
.
.
.
.
...
.
...
....
....
....
..
..
...
...
..
...
...
...
.
...
...
..
...
....
....
..
...
..
.
.
.
.
.
.
.
.
.
..
...
...
..
..
....
....
.
...
..
..
....
...
.....
....
..
..
..
...
..
..
...
..
..
...
...
.
.
.
.
.
.
.
.
.
.
.
...
...
....
....
....
....
...
...
...
...
...
...
...
.... .... ......
...
... ......
... ... ....
.
.
.
.
.
.
.... .. ..
........................................................................................................................................................................
.
....
....
....
.....
.....
......
......
........
.......
.
.
.
.
.
.........
.
.
............
........
...............................................................

15. If p = 2, 2p + p2 = 8 is not prime. If p = 3, 2p + p2 = 17 is prime. If p > 3, then p = 6k 1


for some integer k, so that p2 = 36k 2 12k + 1 is 1 more than a multiple of 3. On the other
hand, when divided by 3, successive powers of 2 leave remainders of 2 and 1 alternately. Since
p > 3 is odd, 2p is 2 more than a multiple of 3. Hence 2p + p2 is divisible by 3, and cannot be
prime. The answer is (b).

16. Let
the other root be t. Then a = t + 2 99. Now b = t(2 99) < 0, so that
t = 99 2 a > 0. Since a is an integer, a 9 2 = 7. The answer is (c).

The Alberta High School Mathematics Competition


Solution to Part I, 2010
1. Since 10 4n 99, 3 n 24. Hence there are 24 3 + 1 = 22 such values. The answer is
(b).
2. There are 7 fences of length 4 and 5 fences of length 6. The total length is 7 4 + 5 6 = 58.
The answer is (a).
3. Since the least common multiple is even, at least one number is even. Since the greatest
common divisor is odd, exactly one number is even. We can show in a similar manner that
exactly one of the two numbers is divisible by 5. Since neither is 10, one of them is 2 and the
other is 5, yielding a sum of 7. The answer is (c).
4. Note that x must be odd, and x = 9 2y
. Since y 0, x 9. Thus there are 5 triples
3
(x, y) = (9, 0), (7,3), (5,6), (3,9) and (1,12). The answer is (b).
5. Note that 2010 54 = 2512.5. There are 502 multiples of 5 from 5 to 2510 inclusive. Hence
2512 is the (2512 502)-th or 2010-th number in the punctured sequence. The answer is (b).
6. Alice and Ethel are 39 floors apart, and as long as the get-together floor is in between, the
total number of floors they cover is 39. Similarly, the total number of floors Brian and Debra
cover is 19. The minimum number of floors Colin covers is 0, when they get together on floor
3. The answer is (e).
7. The pigeons can choose the pigeonholes in 9 9 = 81 ways. There are 12 pairs of rooms
separated by an interior wall. Since the pigeons can choose these two rooms in 2 ways, the
8
desired probability is 212
= 27
. The answer is (d).
81
8. Since x1 3, x 0. Multiplying by x3 , we have 13 x. Hence the set of all values of x is
{ 13 x < 0}. The answer is (d).
9. Since AB = DM and AB is parallel to DM, ABMD is a parallelogram. Similarly, ABCM is
a parallelogram. Therefore, 6 AMD = 6 BCD = 50 and 6 BMC = 6 ADC = 30 . Therefore,
6 AMB = 180 6 AMD 6 BMC = 180 50 30 = 100 . The answer is (c).
A

..................................................................................................................................................
....... ......
...
....
......
....
...
......
......
....
.
...
.
......
.
.
.
...
....
.......
.....
.
.
.
.
.
.
.
.
.
.
.
...
...
...
...
.
.
.
.
.
.
.
...
.
.
.
.
.
...
....
....
....
.
.
.
.
.
.
.
.
.
.
.
.
...
...
....
....
.
.
.
.
.
.
.
.
.
...
.
.
.
...
...
....
.
.
.
.
....
.
.
.
.
.
.
.
... ......
...
.
..
.
.
.
.
.
.
.
.
................................................................................................................................................................................................................................................................

10. We first count the triangles in which the equal sides of length k are longer than the third side,
which can be of length from 1 to k1. Summing from k = 1 to 9, we have 0+1+2+ +8 = 36
such triangles. We now count the triangles in which the equal sides of length k are shorter
than the third side, which can be of length from k + 1 to 2k 1. Summing from k = 1 to 9,
we have 0 + 1 + 2 + 3 + 4 + 3 + 2 + 1 + 0 = 16. The total is 36 + 16 = 52. The answer is (c).

256

512

81

11. The first three choices are equal respectively to 22 , 22 and 22 . Clearly, the second one is
16
the largest among them. The fourth number is equal to 23 . Since 2512 = 4256 > 3256 > 316,
16
the second number is larger than the fourth one. The fifth number is equal to 32 . Clearly,
256
255
255
16
22 = 42 > 32 > 32 . Hence the second number is the largest among the five choices.
The answer is (b).
12. Note that ab + a + b + 1 = (a + 1)(b + 1). Every composite number can be written in this
form and no prime number can be written in this form. Therefore, the positive integers that
are not gold numbers are those that are one less than a prime. By simple counting, we see
there are 8 primes from 2 to 21. Therefore, the number of gold numbers between 1 and 20
inclusive is 20 8 = 12. The answer is (e).
13. The sphere which passes through A, B, C and D also passes through the other four vertices
of a 1 2 2 block having
the space diagonal
A, B, C and D as four of its vertices. Since
3
2
2
2
of this block is of length 2 + 2 + 1 = 3, the radius of the sphere is 2 . The answer is (a).
14. Each application of f doubles the exponent while each application of g quadruples the exponent. After 50 applications of f and 49 applications of g, the exponent has been doubled
50 + 2 49 = 148 times so that n = 2148 . The answer is (c).
15. Denote the area of triangle T by [T ]. Since triangles AXZ and AY C are similar, ZC = 2AZ.
Since triangles AY Z and ABC are similar, Y B = 2AY . It follows that
2
2
2
2
[XY Z] = [AY Z] = [AY C] = [ABC] = .
3
9
27
27
The answer is (b).
A

......
.... ..
.... ....
....
..
.
.
.
..
..
...
.... ....
..
.... ......
.
...
....
.
.
.... ....
..
.
.
.
.
... ...
..
.
.
.
.
.
.... ...
...
.
.
.... ...
.
...
....
....................................................................................
..
.... .......
.
.
..
....
..
.
..
.
...
..
...
....
..
....
.
.
.
..
.
....
..
....
.
.
.
.
..
....
..
.
..
.
....
..
..
.
.
.
....
..
...
.
...
.
.
....
..
.
..
.
.
.
....
..
...
.
.
.
..
....
...
..
.
.
.
...
...
..
.
.
.
.
.
....
..
..
.
.
.
.
..
....
...
..
.
.
.
....
..
..
.
.
.
..
.
...
..
...
.
.
.
.
.
....
..
...
.
.
.
....
.
...
.
.
.
.... ....
..
.
.
.
.
.... ...
...
.
.
.
.... ...
..
.
.
.. ..
.
.
......................................................................................................................................................................................................................................

C
2

and the remainder is 27


. Hence
16. When n3 3n+2 is divided by 2n+1, the quotient is n2 n4 11
8
8
3
2
3
8(n 3n + 2) = (2n + 1)(4n 2n 11) + 27, so that 2n + 1 divides n 3n + 2 if and only if it
divides 27. The set of all values of n for which 2n+1 divides 27 is {14, 5, 2, 1, 0, 1, 4, 13},
and there are 8 such values. The answer is (e).

The Alberta High School Mathematics Competition


Solution to Part I, 2011
1. We have 22012 + 41006 = 22012 + 22012 = 22013. The answer is (a).
2. The surface area of a mini-marshmallow is 6 square centimetres while that of a giant marshmallow is 54 square centimetres. Thus the desired number of mini-marshmallows is 546 = 9.
The answer is (c).
3. Suppose there are m customers on Monday. Then there are 1.2m on Tuesday and 1.5m on
Wednesday. The increase of 0.3m from Tuesday to Wednesday is 25% of 1.2m. The answer
is (b).
4. Sawa is 2 kilometres
southand 2
kilometres west of S. Her distance from S, by the Pythagorean

2
2
Theorem, is 2 + 2 = 8 = 2 2 kilometres from S. The answer is (b).
5. Since 1000 = 2 2 2 5 5 5, the digits can onlybe
 1, 2, 4, 5 and 8. Three of them must
be 5s and they can be placed among the six digits in 63 = 20 ways. The product of the other
three digits is 8, and they are (1,1,8), (1,2,4) or (2,2,2). They can be placed in 3, 6 and 1
ways respectively. Hence the total number of six-digit millenium numbers is 20(3+6+1)=200.
The answer is (e).
6. We have x3 = 24, x4 = 28, x5 = 26 and x6 = 27. The answer is (d).

7. Squaring both sides of 2x2 2x 1 = 2x x2 2x, we have 4x4 8x3 + 4x + 1 = 4x4 8x3,
which simplifies to 4x+1 = 0. Hence the only solution is x = 14 . Indeed, 2( 14 )2 2( 14 ) = 58
q
and 2( 14 ) ( 14 )2 2( 14 ) + 1 = 58 . The answer is (b).
8. Note that g(x) = f (x + 1) f (x) is a linear polynomial. Since g(1) = f (2) f (1) = 2 and
g(2) = f (3) f (2) = 4, we have g(3) = 6. Hence f (4) = f (3) + g(3) = 8 + 6 = 14. The
answer is (b).
9. Since a lucky number n is divisible by 7, it has the form n = 7k for some positive integer k.
If k is not a prime number, then it has a divisor h where 1 < h < k, and 7h is a divisor of n
larger than 7 but not equal to n. Hence k must be a prime number. Moreover, it cannot be
greater than 7. Hence there are only 4 lucky numbers, namely, 14, 21, 35 and 49. The answer
is (d).
10. Annabel spent 15 seconds on each path, and Bethany 18 seconds. On the first path, Bethany
was with Annabel all 15 seconds. On the second path, Bethany joined Annabel 3 seconds late,
and was with her for 12 seconds. On the third path, Bethany was with Annabel for 9 seconds.
On the fourth path, Bethany was with Annabel for 6 seconds. The total is 15+12+9+6=42
seconds. The answer is (b).
11. We can draw a regular polygon of any number of sides such that the side length is 20 centimetres. We can then draw a regular polygon of the same number of sides but with side length
15 centimetres, placed centrally inside the larger polygon. Then a tile can be chosen which
can pave the ring-shaped region inside the larger polygon but outside the smaller one. Hence
the answer is (e).

12. Let x1 , . . . , x20 be the given numbers. If d is the greatest common divisor of these numbers
then


x20
x1
x1 + + x20 = d
+ +
= 462 = 21 22.
d
d
The value d = 22 is obtained if x1 = = x19 = d and x20 = 2d. For each i, xdi 1. Hence
= 23.1. Since d divides 462, the largest value for d is indeed 22. The answer is (b).
d 462
20
13. Dividing throughout by y, we have (z 2 + 1)3 > m(z 3 + 1)2 where z = xy . This is equivalent to
(1 m)z 6 + 3z 4 + (3 2m)z 3 + 1 m > 0
for any positive real z. Hence it is necessary to have 1 m 0, i.e. m 1. If we take m = 1,
the inequality (x2 + y 2)3 (x3 + y 3)2 is equivalent to
x2 y 2((x y)2 + 2x2 + 2y 2) > 0,
which is clearly true. The answer is (c).
 

= 1176 colourings. The number of symmetrical colourings with respect to the


14. There are 49
2
= 24. These colourings are counted twice. All the other colourings are
middle square is 491
2
+ 117624
= 300. The answer is (d).
counted four times. The desired number is 24
2
4
15. Let AD intersect the bisector of 6 C at G. Then 6 CGA = 90 = 6 CGD, 6 GCA = 6 GCD and
GD = GD. Hence triangles GCA and GCD are congruent, so that AC = DC. It follows that
we have BC = 2DC = 2AC. Now among three consecutive positive integers, one is double
another. This is only possible if the integers are 1, 2 and 3, or 2, 3 and 4. The former does
not yield a triangle. Hence AC = 2, AB = 3 and BC = 4, so that AB BC CA = 24. The
answer is (a).
A

....
.......... ....
....... .. ......
....
....... ...
.
.
.
.
.
...
.
....
....
......
...
....
......
..
...
.......................
.
.
.
.
.
.
.
....
......... .
....
.
.
.
.
....
.
.
.
..........
....
.
.
....
.
.
.
.
.
.
.
.
. ............
....
.....
.
.
.
.
.
.
.
.
........
...
....
.
.
.
.
.
.
.
.
.
.
.
....
.........
.
....
.
.
.
.
....
.
.
.
.
.
.
.
.........
.
....
.
.
.
.
.
.
.
.
.
.
.
.
........ ......
.
...
.
.
.
.
......... ....
.
.
.
.
.
....... ..
....
.
.
.
.
.
.
.
.................................................................................................................................................................................................................

c
16. Note that if we write a positive integer m in base 3, then the base 3 representation of b m
3
is simply the base 3 representation of m with the rightmost digit removed. Also, a positive
integer m is divisible by 3 if and only if the rightmost digit of m is 0. Hence, in order that none
of a1, a2, a3 and a4 is divisible by 3, the rightmost four digits of the base 3 representation of
n are all non-zero. Note that 1000 > 2(35 + 34 + 33 + 32 + 3 + 1). If n has at most 6 digits
in its base 3 representation, the first two can be any of 0, 1 and 2, while the last four cannot
be 0. There are 32 24 = 144 such numbers. Clearly, n cannot have more than 7 digits as
otherwise n 37 > 1000. Suppose n has exactly 7 digits. As before, the last four cannot be
0. Since 1000 < 36 + 35 + 33 + 32 + 3 + 1, the first one must be 1, the second must be 0 and
the third can be any of 0, 1 and 2. Hence, there are 3 24 = 48 such numbers. The total is
192. The answer is (b).

The Alberta High School Mathematics Competition


Solution to Part I, 2012
1. Note that 35 + 12 + 40 + 27 = 114 and 1061 = 9 114 + 35. Thus Mr. Sod had spent $35 on
the tenth day of that month at pub A. The answer is (b).
2. Since Meeny played 17 games, Miny and Moe played each other at most 17 + 1 = 18 times,
and each could play at most 18 + 17 = 35 games. As Miny played 35 games, Moe did not
play Meeny but played Miny 18 times. The answer is (a).
3. We want ( Rr )2 to be close to 12 . We have ( 13 )2 < ( 24 )2 < ( 35 )2 < ( 46 )2 =
25
1
1
12 = 98
< 18
= 12 49 , the answer is (e).
49

4
9

and ( 57 )2 =

25
49

. Since

4. Let f (x) = ax2 + bx + c. Then 1 = f (0) = c, 0 = f (1) = a + b + c and 3 = f (2) = 4a + 2b + c.


We have c = 1, a + b = 1 and 2a + b = 1. Hence a = 2 and b = 3, so that f (3) = 10. The
answer is (d).
5. Triangles GAD and GF E are similar, with AD = 3EF . Hence the vertical height of triangle
EF G is 13 of the vertical height of triangle ADG. Hence it is equal to 14 AB = 3 cm so that
the area of triangle EF G is 12 3 4 = 6 cm2. The answer is (a).
A
D
J

JG

B
E
F
C
n(n+1)
6. The sum of the first n positive integers is 2 . Suppose n is even. Then we must have
either n2 = 1 or n + 1 = 1. Both lead to n = 2. Suppose n is odd. Then we must have either
n+1
= 1 or n = 1. However, n = 1 is not allowed by the hypothesis. The answer is (b).
2
3
so that the total number of
7. The fraction of problems solved only by Karla was 45 12 = 10
3
7
= 10
.
problems is a multiple of 10. The fraction of problems solved by Klaus was at most 1 10
Thus the total number of problems was at least 50. If it were 50, then 10 problems were solved
by Klaus alone, and as Karla solved 45 50 = 40 problems, the number of problems solved by
neither is 0. The total number of problems could only be as large as 70 since 35 problems would
be solved by both. In this case, the number of problems solved by neither was 15 70 = 14.
It follows that the total number of problems must be 60, of which 30 were solved by both, 5
3
60 = 18 problems by Karla alone, and 60 30 5 18 = 7 by neither
by Klaus alone, 10
of them. The answer is (d).

8. Note that a > 2 and a < 3 cannot both be true as there are no integers between 2 and 3.
Similarly, a > 4 and a < 5 cannot both be true. Since exactly three of the statements are
true, a < 1 must be true. Hence the largest possible value is a = 0, and for this value, the
three statements a < 1, a < 3 and a < 5 are true and the two statements a > 2 and a > 4
are false. The answer is (a).

9. We have 70 = 1 70 = 2 35 = 5 14 = 7 10. Thus there are four possible shapes of the


rectangle, with respective perimeters 142 cm, 74 cm, 38 cm and 34 cm. The answer is (d).
10. Solving (n + 5)2 < 2n2 < (n + 6)2 yields 50 < (n 5)2 and (n 6)2 < 72. Thus 8 n 5 and
n 6 < 9 or 13 n 14. The answer is (c).
11. We shade the regions AP QB and CRSD while leaving the regions AP SD and BQRC unshaded. Extend the sides of P QRS to the perimeter of ABCD, creating four rectangles at
the corners each of which consists of two congruent triangles, one shaded and one unshaded.
The difference between the total area of the unshaded regions (not counting P QRS) and the
total area of the shaded regions is 1113 363 = 750 cm2. The difference in the lengths of
AD and AB is 15 cm. Hence the side length of P QRS is 750 15 = 50 cm, and the area of
P QRS is 2500 cm2, The answer is (c).
A
D
...............................................................................................................................................................................................................
........................................................................................................................................................................................................................
................................
............................................................................
........................................
............................................................
..................................
...........................................................................
...............................
.............................................
........................
.........................................................................
. ... ...
..........................................
..............................................................
.........................
.............................................
................................
.........................................................................
.......................................
............................................................
..........................
............................................................
................................
............................................................
.......................................
............................................................
..........................
............................................................
.......................................
..............................................................
.........................
..........................................................
................................
............................................................
.......................................
............................................................
..........................
............................................................
......................................
............................................................
...............................
............................................................
..................
............................................................
. ... ...
........................................
............................................................
..........................
.............................................
.........................................................................................................................................................................................................................................................
.................... ..
... .......................................
.......... ...
.
..... .....
.............................................................................................................................................................................................

12. Let the smallest and the largest numbers Weifeng writes down be n2 and m2 respectively.
Since they are the ends of a block of 28 consecutive numbers, (m + n)(m n) = m2 n2 = 27.
We may have m + n = 27 and m n = 1, whereby m = 14 and n = 13. We may have
m + n = 9 and m n = 3, whereby m = 6 and n = 3. Thus the smallest number Weifeng
writes down may be 32 = 9 or 132 = 169. The answer is (e).
13. We have
1
1
1
= 2
+ 2
8
a + 4b + 4 b + 4a + 4
1
1
=
+
(a 2)2 + 4a + 4b (b 2) + 4b + 4a
1
1

+
4(a + b) 4(a + b)
1
=
.
2(a + b)
Hence a + b 4. This maximum value is attained if and only if a = b = 2. The answer is (d).

14. Let M be the midpoint of BC and D the point where the circle is tangent to BC. Let
B 0, C 0 and M 0 be the respective projections of B, C and M on EF . Now AEF is a right
and CC 0 = CE
, so that
isosceles triangle. Hence so are BB 0F and CC 0E. Hence BB 0 = BF
2
2

M M 0 = 12 (BB 0 + CC 0) =

(BF + CE) =
2

(BD + CD) =
2

BC

2 2

2
.
4

The answer is (a).

......
..... .....
...
...
....
...
.
.
.
...
....
...
........................
................................
.
.
.
...
.
........... .....
.......................
..............
.
....... ...
.......................
.
.
.
...
.
..... ...
........................ .......
...
......
.........................
......
.........................
..
...
.......................
.....
..
...
.............................
.........
..
........................
.
0
.
.
.
....
0
........................
.
.. ...
...
.
.
.
.
.......................
.
...
....
.
..
....................
.....
.... ..
...
..
.....
.
.... ....
.
.
.
...
.
.
.. ....
.
...
..
.
.
.
.
.
.
...
.
.
.. ...
.
...
.
..
...
.. ...
..
....
...
..
.. ....
...
....
..
..
.....
..
...
....
....
...
..
..
...
..
....
.
.
.
.
.
.
.
.
....
...
...
.
...
...
...
..
...
..
...
....
...
..
..
...
...
....
...
...
...
...
..
....
...
..
...
..
..
..
....
.
.
.
.
.
.
.
....
.
.
.
.
..
...
..
..
...
..
...
...
..
....
...
...
...
...
...
...
.....
..
...
...
..
...
....
...
...
...
..
..
...
....
.
.
.
.
.
.
.
.
.
.
.
....
.
...
.... ..
.
..
..... ..
... ...
....
...
....
... .
.......
....
.. ......
.
... ..
......
..
.........
... .........
... ....
.............................................................................................................................................................................................................................................................................................................................................

B0
.........
.. .......................

15. Let the numbers of robots and androids be r and a respectively. After one month, these
numbers became r+7a and 7ra. After another month, they became (r+7a)+7(7ra) = 50r
and 7(r + 7a) (7r a) = 50a. Hence after a two-month period, the number of robots became
50 times the original number, and the same goes for the number of androids. There being 6
two-month periods in a year, the initial number of robots was 46875000000 506 = 3 and the
initial number of androids was 15625000000 506 = 1. The answer is (a).
16. Since 13 divides 6(m + 11n) = (6m + n) + 13(5n), 13 divides 6m + n. Since 11 divides
6(m + 13n) = (6m + n) + 11(7n), 11 also divides 6m + n. Hence 11 13 = 143 divides 6m + n,
so that 6m+n = 143k for some integer k. Since 6(m+n) = 143k +5n = 6(24k +n)(k +n), 6
divides k +n so that k +n 6. Now 6(m+n) = 143k +5n = 138k +5(k +n) 138+30 = 168.
Consequently m + n 28, and this is attained if m = 23 and n = 5. The answer is (c).

The Alberta High School Mathematics Competition


Solution to Part I, 2013.
1. Of the first 100 positive integers 1, 2, . . . , 100, the number of those not divisible by 7 is
(a) 14

(b) 15

(c) 85

(d) 86

(e) none of these

Solution:
When 100 is divided by 7, the quotient is 14, with a remainder of 2. Thus 14 of the first 100
positive integers are divisible by 7. It follows that the number of these integers not divisible
by 7 is 100 14 = 86. The answer is (d).
2. The total score of four students in a test is 2013. Ace scores 1 point more than Bea, Bea
scores 3 points more than Cec, and Cec scores 2 points more than Dee. None of their scores
is divisible by
(a) 3

(b) 4

(c) 5

(d) 11

(e) 23

Solution:
Since the average score is just over 500, we try 500 as a score for Dee. Then Cecs score is
502, Beas is 505 and Aces 506. The total is indeed 2013, so that no adjustment is necessary.
Now 500 is divisible by 4, 500 and 505 are divisible by 5, and 506 is divisible by 11 and 23.
The answer is (a).
3. Of the following five fractions, the largest one is
(a)

1
75

(b)

2
149

(c)

3
224

(d)

4
299

Solution:
The lowest common numerator is 12. The fractions then become
respectively. The answer is (b).

(e)

6
449

12
, 12 , 12 , 12
900 894 896 897

and

12
898

4. Two teams A and B played a soccer game on each of seven days. On each day, the first
team to score seven goals wins. There were no ties. Over the seven days, A won more games
than B, but B scored more goals than A overall. The difference in the total numbers of goals
scored by B and A is at most
(a) 17

(b) 18

(c) 19

(d) 20

(e) none of these

Solution:
B won at most three games, and for each of these games, B wins by at most 7 goals with a 7
to 0 score. In the other four games, B loses by at least 1 goal with a 7 to 6 score. The goal
difference is at most 7 3 1 4 = 17. The answer is (a).

5. ABCD is a quadrilateral with AB = 12 and CD = 18. Moreover, AB is parallel to CD


and both 6 ADC and 6 BCD are less than 90 . P and Q are points on side CD such that
AD = AP and BC = BQ. The length of P Q is
(a) 6

(b) 7

(c) 8

(d) 9

(e) 10

Solution:
From A and B, drop perpendiculars onto CD at the points E and F respectively. Since both
6 ADC and 6 BCD are less than 90 , E and F do lie on the segment CD. Note that
DE + F C = CD EF = CD AB = 6.
Since ED = EP and F C = F Q, EP + F Q = 6 < 12 = EF , P is closer to E than Q and Q
is closer to F than P . Therefore, P Q = EF (EP + F Q) = 12 6 = 6. The answer is (a).
A
B







C
 C
 C
 C

C
C
C

D E P

A
 A

A

A

A

A

A

A

6. Each of four cows is either normal or mutant. A normal cow has 4 legs and always lies. A
mutant cow has either 3 or 5 legs and always tells the truth. When asked how many legs they
have among them, their respective responses are 13, 14, 15 and 16. The total number of legs
among these four cows is
(a) 13

(b) 14

(c) 15

(d) 16

(e) none of these

Solution:
Since all four responses are different, at least three of them are wrong. If all four are wrong,
then the cows are all normal, and they will have 16 legs among them. However, this makes
one of the responses right. Hence one of the responses is indeed right. The three normal cows
have 12 legs among them. Hence the mutant cow must have 3 legs in order to make one of
the responses right. The answer is (c).
7. Let a and b be positive integers such that ab < 100 and
a
by m. Then we have
b
(a) m 2.15

a
b

> 2. Denote the minimum value of

(b) 2.15 < m < 2.2

(c) m = 2.2

(d) 2.2 < m < 2.25

(e) m > 2.25

Solution:
We have 100 > ab > b(2b+1) so that b 6. Thus the minimum value of m is 2+ 16 = 2.1666 . . ..
The answer is (b).

8. Let ABCD be a quadrilateral with 6 DAB = 6 CBA = 90 . Suppose there is a point P on


side AB such that 6 ADP = 6 CDP and 6 BCP = 6 DCP . If AD = 8 and BC = 18, the
perimeter of the quadrilateral ABCD is
(a) 70

(b) 72

(c) 74

(d) 76

(e) 78

Solution:
Since 6 DAB + 6 CBA = 180 , AD is parallel to BC. Therefore, 6 ADC + 6 BCD = 180 .
Hence 6 P DC + 6 P CD = 90 . Consequently, 6 DP C = 90 . Let Q be the foot of the
perpendicular on CD from P . Note that triangle P DA is congruent to triangle P DQ, and
triangle P CB is congruent to triangle P CQ. Hence DQ = DA = 8, CQ = CB = 18 and
P A = P Q = P B. Since 6 DP C = 90 , triangle DP Q is similar to triangle P CQ. Hence
DQ
Q
= PQC
. Therefore, P Q2 = 8 18 = 144 and P Q = 12. The perimeter of ABCD is
QP
therefore AP + P B + BC + CQ + QD + DA = 12 + 12 + 18 + 18 + 8 + 8 = 76. The answer
is (d).
C

..........
........ .. ..
......... .... ....
. ..
.........
.
.
.
.
.
.
.
.
.
.
. ...
........
...
.........
....
...
........
..
.........
....
........
..
.
.
.
.
.
.
.
..
.
.
.
......
.
.
.
.
.
.
.
.
...
.
.
......
.
.
.
.
.
.
.
.
.
....
.
.....
.
.
.
.
.
.
.
.
..
.
.
......
.
.
.
.
.
.
.
.
.
...
.
.....
.
.
.
.
.
.
.
.
.
.
.
....
.....
.
.
.
.
.
.
.
.
..
.
.
......
.
.
.
.
.
.
.
.
.
...
.
......
.
.
.
.
.
.
.
.
.
.
...
.
.....
.
.
.
.
.
.
.
.
...
.
.....
.
.
.
.
.
.
.
.
.
.
.
...
...... ...
..
.
.
.
.
.
.
.
.
...
.
..
.....
.
.
.
.
.
.
.
.
.
.
...
.
..
.
......
.
.
.
.
.
.
.
.
.
.
...
.
..
.....
.
.
.
.
.
.
.
.
.
...
.
.
..
.....
.
.
.
.
.
.
.
.
.
.
.
.
...
..
......
..
.
.
.
.
.
.
.
.
.
...
.
..
.
......
.
.
.
.
.
.
.
.
...
.
.
.
..
.
.
..............
.
.
....
..
..
... .......
.
..
.
.
..
.....
...
.
.
...
.
.
.....
..
...
.
.
.
.
.
.
....
.....
...
...
.
.
.
..
.
.
....
...
..
.
.
.
.
.
...
.
.
.....
...
..
.
.
.....
.
...
....
..
.
.....
.
..
...
.
.
..
.....
..
...
..
.
...
.....
...
..
.....
..
.
....
..
.
.....
...
.
..
.
..
.....
.
...
.
.
.
.
...
.
.....
..
.
...
.
.
.
.
...
.....
.
..
...
.
.
.
.
.
...
.
.....
.
..
..
.
.....
....
...
.
.
..
.
.....
.
.
.
..
...
.....
..
.
...
.
.....
.
...
..
..... ....
.
....
...
.
..... .. ...
...
...
..... ... ..
...... ..
...
..
.............................................................................................................................................................................................................................................................................................................

9. Two bus routes stop at a certain bus stop. The A bus comes in one-hour intervals while the
B bus also comes in regular intervals of a different length. When grandma rests on the bench
by the bus stop, one A bus and two B buses come by. Later, grandpa rests on the same bench
and eight A buses come by. The minimum number of B buses that must have come by during
that time is
(a) less than 4

(b) 4 or 5

(c) 6 or 7

(d) 8 or 9

(e) more than 9

Solution:
To minimize the number of B buses coming by, we stretch the length of their intervals as far
as possible. Suppose grandma sees the A bus that comes at 10:00. Then she does not see
the ones that come at 9:00 or 11:00. Thus the length of the interval between two B buses is
strictly less than 2 hours. The two B buses grandma sees may have come at 9:01 and 10:59,
in which case the interval is of length 118 minutes. Grandpa is on the bench for at least 7
hours. This is longer than three intervals for the B buses, so he must have seen at least 3 of
them. Suppose he sees the A buses at 11:00, 12:00, 13:00, 14:00, 15:00, 16:00, 1700 and 18:00.
Then he will see only the B buses at 12:57, 14:55 and 16:53. The answer is (a).

10. Suppose that 162013 = ab , where a and b are positive integers. The number of possible values
of a is
(a) 2

(b) 8

(c) 11

(d) 16

(e) 24

Solution:
Since 162013 = 242013, a must be of the form 2k where k is a positive integer divisor of
4 2013 = 22 3 11 61. The prime factorization of k may contain up to two 2s, one 3,
one 11 and one 61, so that there are (2 + 1)(1 + 1)3 = 24 possible values of k, and therefore
of a. The answer is (e).
11. The following five statements are made about the integers a, b, c, d and e: (i) ab is even and
c is odd; (ii) bc is even and d is odd; (iii) cd is even and e is odd; (iv) de is even and a is odd;
(v) ea is even and b is odd. The maximum number of these statements which may be correct
is
(a) 1

(b) 2

(c) 3

(d) 4

(e) 5

Solution:
If a, c and d are odd while b and e are even, then (i), (ii) and (iv) are all correct. Suppose
at least four statements are correct. By symmetry, we may assume that they are (i), (ii), (iii)
and (iv). However, by (i) and (ii), c and d are both odd, and yet by (iii), cd is even. This is
a contradiction. The answer is (c).
12. A very small cinema has only one row of five seats numbered 1 to 5. Five movie-goers arrive
one at a time. Each takes a seat not next to any occupied seat if this is possible. If not, then
any seat will do. The number of different orders in which the seats may be taken is
(a) 24

(b) 32

(c) 48

(d) 64

(e) 72

Solution:
The first two movie-goers to arrive may take the pair (1,5), (1,4), (2,5), (1,3), (2,4) or (3,5)
of seats. In each case, there are 2!=2 ways for them to do so. If they take (1,5), (1,3) or
(3,5), then the third movie-goer has only one choice of seat. The remaining two seats may be
occupied in 2!=2 ways. Otherwise, the last three movie-goers may take any vacant seats, and
this can be done in 3!=6 ways. Hence the total number of orders in which the seats may be
taken is 2 3 (2 + 6) = 48. The answer is (c).
13. Let f (x) = x2 + x + 1. Let n be the positive number such that f (n) = f (20)f (21). Then the
number of distinct prime divisors of n is
(a) 1

(b) 2

(c) 3

(d) 4

(e) more than 4

Solution:
Note that f (m 1)f (m) = (m2 m + 1)(m2 + m + 1) = m4 + m2 + 1 = f (m2 ). Substituting
m = 21 yields f (20)f (21) = f (441). Therefore, n = 441 = 32 72 . The answer is (b).

14. The number of pairs (x, y) of integers satisfying the equation x2 + y 2 + xy x + y = 2 is


(a) 3

(b) 4

(c) 5

(d) 6

(e) none of these

Solution:
The equation can be written as x2 + (y 1)x + y 2 + y 2 = 0. By the Quadratic Formula,
the solutions are
x=

(y 1)

(y 1)2 4(y 2 + y 2)
2

(y 1)

3(y 1)(y + 3)
2

These are real if and only if (y + 3)(y 1) 0. Since y is an integer, it must be one of 3,
2, 1,0 and 1. If y = 3, then x = 2. If y = 2, then x = 0 or 3. If y = 1, x is not an
integer. If y = 0, then x = 1 or 2. Finally, if y = 1, then x = 0. The answer is (d).
15. A triangle ABC with AB = 7, BC = 8 and CA = 10 has an interior point P such that
6 AP B = 6 BP C = 6 CP A = 120 . Let r1 , r2 and r3 be the radii of the circles passing
through the vertices of triangles P AB, P BC and P CA respectively. The value of r12 + r22 + r32
is
(a) 71

(b) 72

(c) 73

(d) 74

(e) 75

Solution:
Let O be the centre of the circle passing through the vertices of tiangle P AB. Note that since
6 AP B > 90 , O lies on the perpendicular bisector of AB outside of triangle P AB. Since
OA = OP = P B,
AOB = 6 AOP + 6 P OB = 180 26 AP O + 180 26 BP O = 360 26 AP B = 120 .

2
2
. It follows that r2 = AB . Similarly, r2 = BC and
Hence AB = 3OA so that OA = AB
1
2
3
3
3
6

r32 =

CA2
.
3

Hence r12 + r22 + r32 =

72 +82 +102
3

= 71. The answer is (a).

...............................................................
.............
..........
........
........
........
......
......
.
.....
.
.
.
....
...
.
.
.
.....
..
.
.
.
...............
...
.
... .... .......
.
.
... .... .... .............
...
.
.
.
.
.. . . .. ..
..
... ... ... ...........
...
...
...
... ...
.. ...
..
...
... ..
... ...
... ..
...
...
.
.
.
.....
... ....
..
..
.
.
......
.
.
..
..
.....
...
.
.
...
.
..
...
.
.
.
.
.
...
.
..
....
..
..
.
.....
.
.
.
.
.
..
.
..
.. ...
..
.
.
.
.
.
.
.
.. ...
..
..
..
.
....
.. ...
.
.
.
..
..
.. ...
.
.
...
.
.
.
..
.. ...
..
..
.
.
...
.
.
..
...
...
.
..
.
.
...
..
.
..
.
...
.
...
..
..
..
.
.
.
...
..
..
...
..
.
.
...
....
.
.
...
......
.
...
...
.
.
.
.
..
.
...
.. .......
.
...
.
.
.
.
.
.
...
.. .....
..
...
.
.
.
.
.
...
.
.
.
.
.
.
..
......
...
.
...
...
.
.
.
.
.
.
.....
..
...
...
..
.
..
.
.
.
.
.
...
.
.
.....
...
.
..
..
.
.
...
.
.
.
.
.
.
.
.
..... .
...
..
..
...
.
.
.
.
.
.
.
.
...
..
......
..
..
.
..
.
.
.
...
.
.
.
.
.
.
..
..
..
..
...
..
...
... ..........
..
..
.
...
.
..
.
..
.
.
.
...
.
.
.....
.. ..
..
...
..
...
.
.
.
.
...... .. .
..
.
...
...
.
.
.
.
.
.
.
...
.
.......
...
.
..
.
...
.
.
.
.
.
.
.
.
.
..
...
...
...... .................
....
..
.
...
.
.
...
..........
..
.......
....
.
.
.
.
...
.
.
.
.
.
..........
..
..
....
.. ....
...
.
.
.
.
.
.
.
.
.
..........
..
...
...
.. .....
.
.
.
.
.
.. ...
....
.
...
.
.
..........
.. ..
.....
... .....
...
.
.
.
.
.
.
.
.
.
..........
.. ..
...
.....
.. ......
.
.
.
.
.
.
.
.
..........
..
.....
.. .. ...........
.
......
.
.
.
.
.
.
.
.
.......... .....
..................
.......
... .
.
.
.
.........
......................................................................................................................................................................................
.
.
.
.
...........
.
.
.
.
......
.
....................
.
.
.
.
.
.
.
.
.
.
.
.
.
.
.........................

16. The list 1, 3, 4, 9, 10, 12, 13, . . . contains in increasing order all positive integers which can
be expressed as sums of one or more distinct integral powers of 3. The 100-th number in this
list is
(a) 981

(b) 982

(c) 984

(d) 985

(e) 999

Solution:
If we switch the powers of 3 to the powers to 2, then we get all the positive integers. Hence
we convert 100 into base 2, obtaining 100 = 26 + 25 + 22 . It follows that the 100-th number
on the list is 36 + 35 + 32 = 981. The answer is (a).

The Alberta High School Mathematics Competition


Solution to Part I, 2014.
Question 1.
When the repeating decimal 0.6 is divided by the repeating decimal 0.3, the quotient is
(a) 0.2

(b) 2

Solution:
We have 0.6 0.3 =

2
3

(c) 0.5

1
3

(d) 0.5

(e) none of these

= 2.The answer is (b).

Question 2.
The number of two-digit positive integers such that the sum of the two digits is 12 is
(a) 5

(b) 6

(c) 7

(d) 8

(e) 9

Solution:
There are 7 such numbers, namely 39, 48, 57, 66, 75, 84 and 93. The answer is (c).
Question 3.
Let a > b > 0 be prime numbers. Of the following five numbers, the one which cannot be equal to
a b is
(a) 41

(b) 42

(c) 43

(d) 44

(e) 45

Solution:
We have 43 2 = 41, 47 5 = 42, 47 3 = 44 and 47 2 = 45. In order for a b = 43, we must
have b = 2 but then a = 45 is not a prime number. The answer is (c).
Question 4.
Chau and Matt are picking berries. On Monday Matt picks twice as many kilograms of berries as
Chau does, and on Tuesday Chau picks twice as many kilograms of berries as Matt does. Between
them, they pick a total of 30 kilograms of berries over the two days. The number of kilograms of
berries Chau picks over the two days is
(a) 14

(b) 15

(c) 16

(d) some number greater than 16

(e) not uniquely determined


Solution:
Let Chau pick x kilograms of berries on Monday and Matt pick y kilograms of berries on Tuesday.
Then 3x + 3y = 30 so that x + y = 10. The number of kilograms Chau picks over the two days is
x + 2y. This expression can take on infinitely many values if all we know is that x + y = 10. For
example we may have x = 0 so that y = 10 and x + 2y = 20; or we may have x = 4 so that y = 6
and x + 2y = 16; and so on. The answer is (e).

Question 5.
For every set of five of the numbers 1, 2, . . . , 2014, Lac writes down the smallest of the five numbers.
The largest number she writes down is
(a) 5

(b) 2009

(c) 2010

(d) 2014

(e) none of these

Solution:
The largest minimum occurs when Lac chooses the numbers 2010, 2011, 2012, 2013 and 2014, and
writes down 2010. The answer is (c).
Question 6.
The sum of four consecutive integers is n. The largest of these four numbers is
(a) n

(b)

n
4

(c)

n2
2

(d)

n+6
4

Solution:
Let n = (k 3) + (k 2) + (k 1) + k = 4k 6. Then k =

(e) none of these

n+6
.
4

The answer is (d).

Question 7.
Let f (n) = 2n3 for any positive integer n. For any odd number m 3, the largest positive integer
k such that 2k divides f (f (f (m))) is
(a) 4

(b) 8

(c) 12

(d) 13

(e) not uniquely determined

Solution:
We have f (m) = 2m3 , f (2m3 ) = 24 m9 and f (24 m9 ) = 213 m27 . The answer is (d).
Question 8.
Let f be a function such that f (x) + 3f ( x1 ) = x2 for any nonzero real number x. Then the value of
f (3) is
(a) 77
72

(b) 37
36

25
(c) 24

(d) 13
12

(e) none of these

Solution:
Replacing x by -3 and then by 13 in the given equation, we obtain f (3) + 3f ( 31 ) = 9 and
13
respectively f ( 13 ) + 3f (3) = 19 . Solving for f (3) we get f (3) = 12
. The answer is (d).
Alternative Solution: Replacing x by x1 in the given equation, we obtain f ( x1 ) + 3f (x) =
4
39
for f (x) we get f (x) = 3x
. Hence f (3) = 36
= 13
.
8x2
12

1
.
x2

Solving

Question 9.
The number of integer pairs (m, n) such that mn = m + n is
(a) 0

(b) 1

(c) 2

(d) 3

(e) more than 3

Solution:
n
1
Clearly n 6= 1, so that m = n1
= 1 + n1
. Since m is an integer, we can only have n = 0 or 2,
with m = 0 and 2 respectively. The answer is (c).

Question 10.
The number of five-digit positive integers such that the digits are alternately odd and even (either
odd-even-odd-even-odd or even-odd-even-odd-even) is
(a) 10 55

(b) 2 55

(c) 55

(d) 9 54

(e) none of these

Solution:
For the odd-even-odd-even-odd pattern, we have 55 such numbers. For the even-odd-even-odd-even
pattern, we have 4 54 such numbers. The total is 9 54 . The answer is (d).
Question 11.
Let a, b and c be real numbers such that a + b + c = 5 and
a
b
c
+ c+a
+ a+b
is
b+c
(a) 21

(b) 23

Solution:
Note that

Hence

a
b+c

(c) 25

(d) 27

1
b+c

1
c+a

1
a+b

= 6. The value of

(e) not uniquely determined

a
b
c
5 (b + c) 5 (c + a) 5 (a + b)
+
+
=
+
+
b+c c+a a+b
b+c
c+a
a+b
5
5
5
=
+
+
3.
b+c c+a a+b
+

b
c+a

c
a+b

= 5 6 3 = 27. The answer is (d).

Question 12.
P is a point inside an acute triangle ABC. D, E and F are the feet of the perpendiculars from
P on BC, CA and AB respectively. If BD = 1, DC = 10, CE = 6, EA = 9 and AF = 13, the
length of F B is
(a) 1

(b) 2

(c) 3

(d) 4

(e) 5

Solution:
By Pythagoras Theorem, BD2 DC 2 = (BP 2 P D2 ) (CP 2 P D2 ) = BP 2 CP 2 . Similarly,
CE 2 EA2 = CP 2 AP 2 and AF 2 BF 2 = AP 2 BP 2 . Summing these equations yields
(BD2 + CE 2 + AF 2 ) (DC 2 + EA2 + F B 2 ) = 0. Hence F B 2 = (12 + 62 + 132 ) (102 + 92 ) = 52
so that F B = 5. The answer is (e).
Question 13.
Three candles which can burn for 30, 40 and 50 minutes respectively are lit at different times. All
three candles are burning simultaneously for 10 minutes, and there is a total of 20 minutes in which
exactly one of them is burning. The number of minutes in which exactly two of them are burning
is
(a) 35

(b) 45

(c) 70

(d) 90

(e) none of these

Solution:
Count the number of minutes in which each candle is burning and add them together. The ten minutes of simultaneous burning contributes 30 minutes to this, and there are 20 minutes of individual
burning. The total must be 30+40+50=120 minutes, so the time when exactly two candles are
burning must contribute the remaining 120-30-20=70 minutes. Thus the answer must be 70/2=35.
Alternative Solution: Let the first two candles burn simultaneously for a minutes, the first and
the third for b minutes and the last two for c minutes. Then the first candle burns alone for
20 a b minutes, the second for 30 a c minutes and the third for 40 b c minutes. Hence
90 + 2(a + b + c) = 20 so that a + b + c = 35. This may be realized if a = 0, b = 20 and c = 15.
The three candles are lit at minutes 30, 0 and 15 respectively. The answer is (a).
Question 14.
P is a point inside a convex quadrilateral ABCD of area 168 such that P A = 9, P B = P D = 12
and P C = 5. The perimeter of the quadrilateral is
(a) 38

(b) 56

(c) 58

(d) 60

(e) 62

Solution:
Note that AC AP + P C = 14, BD BP + P D = 24 and the area of ABCD is at most
1
AC BD 168. Thus we must have equality, so that P is the point of intersection of the diagonals
2

which must be perpendicular


to each other. By Pythagoras Theorem, AB = AD = 92 + 122 = 15

and CB = CD = 52 + 122 = 13. Hence the perimeter of ABCD is 2(15 + 13) = 56. The answer
is (b).
Question 15.
For any real numbers x and y, the minimum value of x4 5x2 + y 2 + 2x + 2y + 2xy + 6 is
(a) 5

(b) 4

(c) 1

(d) 0

(e) none of these

Solution:
2
2
2
2
2
Since x4
5x + y + 2x + 2y+ 2xy + 6 = (x 3) + (x + y + 1) 4, minimum value is 4, attained
at x = 3 and y = 1 3. The answer is (b).
Question 16.
Four cages are arranged in a 2 2 formation. Each cage contains some chickens and some rabbits
(some cages may contain only chickens or only rabbits). The total number of heads in the two cages
in the first row is 60. The total number of legs in the two cages in the second row is 240. The total
number of heads in the two cages in the first column is 70. The total number of legs in the two
cages in the second column is 230. The minimum number of animals in all four cages is
(a) 120

(b) 128

(c) 145

(d) 180

(e) none of these

Solution:
There are 230 legs in the two cages in the second column. Since 230 = 4 57 + 2, they may
come from as few as 58 animals, namely, 57 rabbits and 1 chicken. Hence the minimum number of
animals in all four cages is 70+58=128. If we have 55 animals in the first cage in the first row, 0

chickens and 5 rabbits in the second cage in the first row, 15 chickens and 0 rabbits in the first cage
in the second row, and 1 chicken and 52 rabbits in the second cage in the second row, we have 128
animals overall. The answer is (b).

The Alberta High School Mathematics Competition


Solution to Part I, November 2015.
Question 1.
How many three digit numbers have the product of the three digits equal to 5?

(a) 1

(b) 2

(c) 3

(d) 5

(e) 6

Solution:
The numbers are 115, 151, 511. The answer is (c).
Question 2.
Let m, n be positive integers such that 230 330 = 8m 9n . Determine the value of m + n.

(a) 15

(b) 20

(c) 25

(d) 30

(e) 35

Solution:
The equation can be written as 2303m = 32n30 hence m = 10, n = 15, thus m + n = 25. The answer is (c).
Question 3.
The x-intercept, y-intercept, and slope of a certain straight line are three nonzero real numbers. The number of
negative numbers among these three numbers is:
(a) 0 or1

(b) 1 or 2

(c) 2 or 3

(d) 0 or 2

(e) 1 or 3

Solution:
The slope of the line with the x- intercept at (a, 0) and y-intercept at (b, 0) is m = ba . If a, b are of the same sign, m
is negative and if they are of opposite sign m is positive. Hence the number of negative numbers among a, b, m is
1 or 3. The answer is (e).
Question 4.
The length of a certain rectangle is increased by 20% and its width is increased by 30%. Then its area is increased
by:
(a) 25%

(b) 48%

(c) 50%

(d) 56%

(e) 60%

Solution:
If l and w denote the length and width of the rectangle then its area is A = l w while the area of the increased
rectangle is
20l
30w
156
156
56
(l +
) (w +
)=l w
= A
= A+A
.
100
100
100
100
100
Thus the area of the rectangle is increased by 56%. The answer is (d).
Question 5.
Each of Alan, Bailey, Clara and Diane has a number of candies. Compared with the average of the number of candies each person has, Alan has 6 more than the average, Bailey has 2 more than the average, Clara has 10 fewer
than the average and Diane has k candies more than the average. Determine k.

(a) 1

(b) 2

(c) 3

(d) 4

Page 1 of 5

(e) not uniquely determined

Solution:
Let m be the average in question. Then the four people have a total of 4m candies. The Alan, Bailey, Clara has
m + 6, m + 2, m 10 candies, which totals 3m 2 candies. Therefore, Diane has 4m (3m 2) = m + 2 and thus has
2 more candies than the average.
An alternate approach: since the total of the differences from the average must be zero, Diane should have just
10-6-2=2 candies more than the average.
The answer is (b).
Question 6.
Ellie wishes to choose three of the seven days (Monday, Tuesday,. . . , Sunday) on which to wash her hair every week,
so that she will never wash her hair on consecutive days. The number of ways she can choose these three days is:
(a) 6

(b) 7

(c) 8

(d) 10

(e) 14

Solution:
Ellie can choose on of the following triplets: (M, W, F), (M, W, Sa), (M, R, Sa), (T, R, Sa), (T, R, Su), (T, F, Su), (W, F,
Su). There are seven possibilities.
Here is an alternate solution: In any such choice of three wash days, exactly one of them must be followed by
two non-wash days. The choice of this day will determine the other two wash days. There are seven possibilities
and thus the answer is (b).
Question 7.
How many different sets of two or more consecutive whole numbers have sum 55?
(a) 2

(b) 3

(c) 4

(d) 5

(e) none of these

Solution:
The sum of k positive consecutive integers is
a + (a + 1) + (a + k 1) = ka +

k(k 1) k(2a + k 1)
=
.
2
2

and thus k(2a + k 1) = 110 = 2 5 11. The solutions (k, a) are (2, 27), (5, 9), and (10, 1) for which
55 = 27 + 28 = 9 + 10 + 11 + 12 + 13 = 1 + 2 + + 10.
If one consider the set of whole numbers W = {1, 2, 3, ...} then there are three sets of consecutive whole numbers
having the sum 55. However, if W = {0, 1, 2, 3, ...} then also
55 = 0 + 1 + 2 + + 10.
and we find four sets with the required property.
The answer is (b) or (c).
Question 8.
There are 5 boys and 6 girls in a class. A committee of three students is to be made such that there is a boy and a
girl on the committee. In how many different ways can the committee be selected?
(a) 100

(b) 135

(c) 145

(d) 155

(e) 165

Solution:

The number of committees of 3 students made with 11 students is 11
3 = 165. The number of committees of three
6 5
girls or three boys is 3 + 3 = 20 + 10 = 30. The number of requested committees is 165-30=135. The answer is (b).
Question 9.
In a class with 20 students, 14 wear glasses, 15 wear braces, 17 wear ear-rings and 18 wear wigs. What is the
minimum number of students in this class who wear all four items?
Page 2 of 5

(a) 4

(b) 6

(c) 7

(d) 9

(e) 10

Solution:
We have 6 students not wearing glasses, 5 students not wearing braces, 3 students not wearing ear-rings and 2
students not wearing wigs. Even if these are 6+5+3+2=16 different students, we still have 20 16 = 4 students
wearing all four items. The answer is (a).
Question 10.
Each person has two legs. Some are sitting on three-legged stools while the others are sitting on four-legged chairs
such that all the stools and chairs are occupied. If the total number of legs is 39, how many people are there?
(a) 5

(b) 6

(c) 7

(d) 8

(e) 9

Solution:
Let the number of stools be m and the number of chairs be n. Then 5m + 6n = 39. Hence m is a multiple of 3 but
not a multiple of 6. Moreover, 5m < 39 so that m 7. It follows that m = 3 and n = (39 3 5) 6 = 4, so that the
number of people is 3+4=7. The answer is (c).
Question 11.
The number of integers n for which the fraction

(a) 503

(b) 504

22015
5n+1

(c) 1006

is an integer is

(d) 1007

(e) 1008

Solution:
We should have 5n +1 = 2k where 0 k 2015. If 5n +1 = 2k , then 5|(2k 1), which happens if k = 0, 4, 8 , hence
k = 4s, 0 s 503. For these values of k we obtain 504 nonnegative integers n. If 5n +1 = 2k then 5|(2k +1), which
happens if k = 2, 6, 10, , hence k = 4s + 2, 0 s 503. For these values of k we obtain 504 negative integers n.
Therefore there are 1008 convenient values for n. The answer is (e).
Question 12.
In the diagram below, which is not drawn to scale, the circles are tangent at A, the centre of the larger circle is at O
and the lines AB and C D are perpendicular.

C
F
A

If E B = 3 and F C = 2 then the radius of the smaller circle is


(a) 4/3

(b) 5/3

(c) 5/2

(d) 3

Page 3 of 5

(e) 7/2

Solution:
Let R, r denote the lengths of the large and respectively the small radius and E B = a, F C = b. First a = 2R 2r so
a2
a
that R = r + a/2. If O 0 denotes the centre of the smaller circle then O 0 F 2 = O 0 O 2 +OF 2 hence r 2 =
+ (r + b)2 .
4
2
(2b a)2 + a 2
5
Solving for r we get r =
. Taking a = 3, b = 2 we get r = . The answer is (c).
4(2b a)
2
Question 13.
Consider the expansion
3

1 + x + x 2 + + x 50 = c 0 + c 1 x + c 2 x 2 + + c 150 x 150 .
The value of the coefficient c 50 is
(a) 1274

(b) 1275

(c) 1326

(d) 1378

(e) none of these

Solution:

3
1 + x + x 2 + + x 50

= 1 + x + x 2 + + x 50 1 + x + x 2 + + x 50 1 + x + x 2 + + x 50
The coefficient of x 50 is just the number of x a x b x c = x a+b+c with a + b + c = 50, a, b, c {0, 1, , 50}. If a = 0 the
equation b + c = 50 has 51 solutions, namely (0, 50) , (1, 49) , , (50, 0) . Also, if a = 1, the equation b + c = 49 has 50
solutions and so on. The number of all solutions is
51 + 50 + + 1 = 1326
Hence c 50 = 1326. The answer is (c).
Question 14.
A 1000 digit number has the property that every two consecutive digits form a number that is a product of four
prime numbers. The digit in the 500th position is
(a) 2

(b) 4

(c) 5

(d) 6

(e) 8

Solution:
The numbers of two digits which are written as product of four prime numbers are the following: 2 2 2 2 =
16, 2 2 2 3 = 24, 2 2 2 5 = 40, 2 2 2 7 = 56, 2 2 2 11 = 88, 2 2 3 3 = 36, 2 2 3 5 = 60, 2 2 3 7 = 84, 2 3 3 3 =
54, 2 3 3 5 = 90, 3 3 3 3 = 81.
We conclude that the number that satisfies the conditions in the problem should have all its digits equal to 8. The
answer is (e)
Question 15
Points E and F are on the sides BC and respectively C D of the parallelogram ABC D such that
Let M be the intersection of AE and B F . The value of

(a) 11

(b) 11 12

(c) 12

AM
is equal to
ME

(d) 12 12

Solution:

Page 4 of 5

(e) 12 34

EB 2
FC
1
= and
= .
EC 3
FD 4

The parallel line to F B through C intercepts AB at P and AM intercepts C P at Q. Then


MQ M E + EQ
EQ
EC
3 5
=
= 1+
= 1+
= 1+ =
ME
ME
ME
EB
2 2
and

Hence

AM
AB DC DF + F C
DF
=
=
=
= 1+
= 4+1 = 5
MQ B P
FC
FC
FC
AM
ME

AM
MQ

MQ
ME

= 5 25 = 12.5. The answer is (d).

Question 16.
Each of Alvin, Bob and Carmen spent five consecutive hours composing problems. Alvin started alone, and was
later joined by Bob. Carmen joined in before Alvin stopped. When one person was working alone 4 problems were
composed per hour. When two people were working together, each only composed 3 problems per hour. When all
three were working, each composed only 2 problems per hour. No coming or going occurs during the composition
of any problem. At the end, 46 problems were composed. How many were composed by Bob?
(a) 14

(b) 15

(c) 16

(d) 17

(e) 18

Solution:
The total work period may be divided into five intervals by comings and goings. The respective numbers of people
working during these intervals are 1, 2, 3, 2 and 1 respectively. Note that the total length of any three consecutive
intervals is 5 hours. Hence the fourth interval has the same length as the first and the fifth interval has the same
length as the second. During each of the second, third and fourth interval, the number of problems composed was
6 per hour since 3+3=6=2+2+2. Hence 5 6 = 30 problems were composed when Bob was working. The number
of problems composed when Alvin or Carmen was working alone was 46-30=16. It follows that the total length of
these two intervals is equal to 16 4 = 4 hours. Hence the total length of the second and the fourth interval is also
4 hours, so that the length of the third interval is 5 4 = 1 hour. Thus the number of problems composed by Bob
was 1 2 + 4 3 = 14. The answer is (a).

Page 5 of 5

Alberta High School Mathematics Competition


Solutions and Comments to the Second Round, 2007.
Problem 1.
This problem really consists of two parts, finding values for n and proving that there are no more.
Most contestants got somewhere with the first part, but many faltered in the second.
For n = 1, 2, 3 and 4, there are no positive integers m such that m2 + 4 n. Hence these four
values have the desired property vacuously. While not an essential part of the problem, these values
should be included for completeness.
If the maximum value of m is 1, then 12 + 4 n < 22 + 4 and n = 5, 6 or 7. Since 1 divides
all of them, these three values have the desired property. If the maximum value of m is 2, then
22 + 4 n < 32 + 4 and n = 8, 9, 10, 11 or 12. Of these, only 8, 10 and 12 are divisible by both 1
and 2. If the maximum value of m is 3, then 32 + 4 n < 42 + 4 and n = 13, 14, 15, 16, 17, 18
or 19. Of these, only 18 is divisible by all of 1, 2 and 3. If the maximum value of m is 4, then
42 + 4 n < 52 + 4 and n = 20, 21, 22, 23, 24, 25, 26, 27 or 28. Of these, only 24 is divisible by
all of 1, 2, 3 and 4. It will turn out that no positive integers other than 1, 2, 3, 4, 5, 6, 7, 8, 10, 12,
18 and 24 have the desired property.
For the second part, Jeffrey Mo of William Aberhart High School argued as follows. Suppose the
maximum value of m is k for some integer k 5. Then k 2 + 4 n < (k + 1)2 + 4. In order for n to
be divisible by just k 1 and k, it has to be a multiple of k(k 1) since k 1 and k are relatively
prime. Now k(k 1) < k 2 + 4 while 2k(k 1) ((k + 1)2 + 4) = k 2 4k 5 = (k + 1)(k 5) 0
for k 5. Hence n cannot be a multiple of k(k 1), so that there are no solutions for m 5.
Jerry Lo of Ross Sheppard High School argued as follows. Suppose we have solutions n for some
m 5. Then n must be a multiple of m. Now m2 +4 < m2 +m < m2 +2m < (m+1)2 +4 m2 +3m,
with equality holding in the last case only for m = 5. If n = m2 + 3m, then we must have m = 5
so that n = 40, but 40 is not divisible by 3. Hence n = m(m + 1) or m(m + 2). Note that n must
also be a multiple of m 1. Note that m 1 and m are relatively prime. If n = m(m + 1), then
m 1 must divide m + 1 = (m 1) + 2. Hence it must divide 2, so that m 3. If n = m(m + 2),
then m 1 must divide m + 2 = (m 1) + 3. Hence it must divide 3, so that m 4. Either case
contradicts m 5. Hence there are no solutions n for m 5.
Problem 2.
Far too many contestants did not know that the total number of tables is 15!. For those who did,
the majority merely observed that the maximum sum of a row is 13+14+15=42 and the minimum
sum is 1+2+3=6. From these, they concluded that the total of the two sums must be 48, in that
if the maximum sum drops, the minimum sum would rise and compensate. While this may be a
loose description of what is the case, it did not explain why this is the case. The argument really
rests on one simple fact. The following solution by Linda Zhang of Western Canada High School
is typical of those of the top contestants.
For each table A, there is a table B which may be obtained from A by subtracting each number in
A from 16. Note that A and B are distinct tables. Now the row in A with the largest sum turns
into the row in B with the smallest sum, and the row in A with the smallest sum turns into the row
in B with the largest sum. The largest row sum of A plus the smallest row sum of B is 48, as is the
largest row sum of B plus the smallest row sum of A. Since the 15! tables may be divided into 15!
2
such pairs, the sum of the 2 15! numbers on our record is 48 15!.

Problem 3.
This turned out to be the problem in which most contestants could make some progress. However,
many approached it haphazardly, and managed to find only some of the answers. Others found
all the answers but did not prove that there are no more. We give the solution by Jarno Sun of
Western Canada High School.
Let ABC be the triangle. Let 6 ABC = 36 . We may assume that 6 CAB 6 BCA. Then

6 CAB 180 36 = 72 > 6 ABC. In order for ABC to be divided into two triangles with a
2
straight cut, the cut must pass through a vertex. We consider three cases:
Case 1. The cut passes through B.
Let the cut meet CA at E. Since 6 CAB > 6 ABC > 6 ABE, 6 BEA must be one of the equal
angles in triangle BEA. It follows that 6 BEA is acute so that 6 BEC is obtuse. (See the diagram
below.) Let 6 EBC = 6 BCA = x . Then 6 BEA = 2x. We consider two subcases:
Subcase 1a. 6 BEA = 6 CAB.
Then 6 ABE = 180 4x and 36 = 6 ABC = (180 4x ) + x. This yields x = 48 but then
6 ABE = 12 . This is impossible.
Subcase 1b. 6 BEA = 6 ABE.
Then 6 ABE = 2x and 36 = 6 ABC = 2x + x . This yields x = 12. It follows that ABC is a
(132 , 36 , 12 ) triangle.
A
...
... ..............................
....
................
....
................ E
....
.
.
.
.
.
.........................
................
...
.............
................
....
...............
................
....
................
................
....
................
................
.... .............................
.
................
.
.
......
.......................
.
.
.
.
.
.. ............................................................................................................................................................................................................................................................

Case 2. The cut passes through A.


Let the cut meet BC at D. We consider three subcases:
Subcase 2a. 6 BDA = 6 ABC = 36

Then 6 ADC is obtuse. (See the first diagram below.) We must have 6 BCA = 6 CAD = 362 = 18 ,
so that ABC is a (126 , 36 , 18 ) triangle.
Subcase 2b. 6 BAD = 6 ABC = 36 .

= 54 and 6 CAB = 90 . (See


Then 6 BDA = 108 . If AD = CD, then 6 DAC = 6 BCA = 108
2

the second diagram below.) It follows that ABC is a (90 , 54 , 36 ) triangle. If AD = AC, then
6 BCA = 6 ADC = 72 and 6 CAB = 180 36 72 = 72 . (See the third diagram below.) It
follows that ABC is a (72 , 72 , 36 ) triangle. Finally, if AC = CD, then 6 CAD = 6 ADC = 72 .
Hence 6 BCA = 36 and 6 ABC = 108 . (See the fourth diagram below.) It follows that ABC is a
(108 , 36 , 36 ) triangle.
Subcase 2c. 6 BAD = 6 BDA = 72 .

Then 6 ADC is obtuse. We must have 6 BCA = 6 CAD = 722 = 36 and ABC is again a
(108 , 36 , 36 ) triangle.
A
A
A
A
....
...
..
.......
.... ...............
..........
.........
....... ....
..... ...........
...
.... ...........
....
....
...........
....
...........
.....
....
...........
....
.....
..........
.
.
....
.
...........
....
...
.
.
...........
.
.....
...
..
.
.
.
.
.....................................................................................................................................................................

... . ..
.... ... .....
..... ....
...
...
.....
.
...
....
.
.
.
...
...
...
..
.
.
.
.
...
...
..
.
.
.
..............................................................................

.
.... .. ..
.... ... ...
.... .... ...
.....
..
.
...
.
.
..
.
.
.
..
..
....
.
.....
...
.................................................................

..... .. ........
.... ..
....
.... ....
.....
....
.
....
....
.
..
....
.
.
..
....
...
.
.
.
.
.
....
.
..
.
.
.
.
.
.
.....................................................................................................

D C

Case 3. The cut passes through C.


Let the cut meet AB at F . Since 6 CAB 6 BCA > 6 ACF , 6 AF C must be one of the equal
angles in triangle AF C. It follows that 6 CF B is obtuse. It follows that 6 BCF = 6 ABC = 36
and 6 AF C = 72 . Since 6 CAB 6 BCA, ABC is again a (72 , 72 , 36 ) triangle.
In summary, the largest angle of ABC, namely 6 CAB, may be 72 , 90 , 108 , 126 or 132 .

Problem 4.
With greater reliance on graphing calculators and computer software, the majority of students
nowadays are very uncomfortable with algebraic manipulations. A problem such as this has become
inaccessible to most contestants.
Brett Baek of Western Canada High School used the following approach.
3

From 1a
= 1b
, we have b a3b = a ab3. Hence a b = ab(b2 a2) so that ab(a + b) = 1.
a
b
Similarly, bc(b + c) = 1. From these two equations, we have a2 c2 = bc ab or (a + c)(a c) =
b(a c). Since a 6= c, we have a + b + c = 0. Hence abc(a + b) = c = a + b. Since a + b + c = 0
but c 6= 0, a + b 6= 0 and we have abc = 1. Now
0 =
=
=
=

(a + b + c)3
a3 + 3a2 (b + c) + 3a(b + c)2 + (b + c)3
(a3 + b3 + c3) + 3(a + b + c)(bc + ca + ab) 3abc
(a3 + b3 + c3) + 0 3.

It follows that the only possible value of a3 + b3 + c3 is 3.


For those with more knowledge of algebra, this problem was practically trivial. Jerry Lo had the
most succinct write-up.
The given conditions show that a, b and c are roots of the equation x3 + kx 1 = 0 where k is the
common value of the three fractions. Hence
x3 + kx 1 = (x a)(x b)(x c)
= x3 (a + b + c)x2 + (bc + ca + ab)x abc.
It follows that a+b+c = 0, abc = 1 and a3 +b3 +c3 = (a+b+c)(a2 +b2 +c2 bccaab)+3abc = 3.
Problem 5.
This problem, which looks deceptively easy, is very annoying. Only one contestant gave a complete
argument, and a handful of others came close. There is a relatively easy part of the problem that
many contestants got, that is, showing that c = 50. Suppose z people in all responded to the survey.
z
c
Then 2006
= 100
or 50z = 1003c. Since 50 and 1003 are relatively prime, c must be a multiple of 50.
Since we are given that 0 < a < c < b < 100, the only possible value is c = 50. After this, things
get messy. What follows is the approach by Jeffrey Mo.
Let the total number of teachers be d and the number of those teachers who responded be x.
a
b
Then xd = 100
and 1003x
= 100
. From the first, we have ad = 100x. From the second, we have
2006d
2006b bd = 100300 100x = 100300 ad. This may be rewritten as (b a)d = 2006(b 50). It
follows that 1003 = 17 59 divides (b a)d. Now b a < 100 < 1003 and we also have d < 1003
since a < c = 50. Hence there are two cases.
Case 1. 59 divides b a.
This means of course that b a = 59 and d = 2 17(b 50). Hence 50x = ad
= 17a(a + 9). Since
2
25 is relatively prime to 17 and to at least one of a and a + 9, it must divide either a < 50 or
a + 9 < 59. We consider three subcases.
Subcase 1a. a = 25.
= 289 and d = 2 17(84 50) = 1156. Hence
We have b = 25 + 59 = 84, x = 1725(25+9)
50
1003 289 = 714 and 2006 1156 = 850. It follows that 289 of 1156 teachers and 714 of 850
students responded to the survey.

Subcase 1b. a + 9 = 25.


We have a = 25 9 = 16, b = 16 + 59 = 75, x = 171625
= 136 and d = 2 17(75 50) = 850.
50
Hence 1003 136 = 867 and 2006 850 = 1156. It follows that 136 of 850 teachers and 867 of 1156
students responded to the survey.
Subcase 1c. a + 9 = 50.
We have a = 50 9 = 41 and b = 41 + 59 = 100. This contradicts b < 100 and there are no
solutions in this subcase.
Case 2. 17 divides b a.
Then ba = 17n where n 5. We have nd = 259(b50). Hence 50nx = and
= 59a(17n50+a).
2
We consider five subcases, none of which yields additional solutions.
Subcase 2a. n = 1.
We have 50x = 59a(a 33) and 25 must divide either a or a 33. The former means a = 25,
but then a 33 < 0. The latter means a 33 25, but then a 58 > 50 = c. Both lead to
contradictions.
Subcase 2b. n = 2.
We have 100x = 59a(a 16) and 25 must divide either a or a 16. The former means that a = 25,
but then 59a(a 16) is odd. The latter means a 16 = 25, but then 59a(a 16) is again odd.
Subcase 2c. n = 3.
We have 150x = 59a(a + 1) and 25 must divide either a or a + 1. The former means that a = 25,
but then 59a(a + 1) is not divisible by 3. The latter means a + 1 = 25 or 50. If a = 49, 59a(a + 1)
is again not divisible by 3. If a = 24, then x = 236, but then d = 100236
is not an integer.
24
Subcase 2d. n = 4.
We have 200x = 59a(a + 18) and 25 must divide either a or a + 18. The former means that a = 25,
but then 59a(a + 18) is odd. The latter means a + 18 = 25 or 50. If a = 7, 59a(a + 18) is again
odd. If a = 32, then b = 32 + 68 = 100, and this contradicts b < 100.
Subcase 2e. n = 5.
We have 250x = 59a(35+ a) and 25 must divide either a or 35+ a. However, this means that a 25
and b = a + 85 > 100, a contradiction.
A shorter approach goes as follows. Let s and t be the respective numbers of students and teachers in
ta
sb
ta
sb
the survey. Then s+t = 2006, both 100
and 100
are integers, and 100
+ 100
= 1003. Note that 5 cannot
divide both s and t. If 5 does not divide s, then b must be a multiple of 25. Since 50 < b < 100, we
must have b = 75. If 5 does not divide t, then a must be a multiple of 25. Since 0 < a < 50, we must
have a = 25. If 5 does not divide either s or t, then a = 25 and b = 75 and we have t + 3s = 4012.
Subtract from this s + t = 2006 and we have 2s = 2006, so that s = t = 1003. However, neither
ta
sb
nor 100
is an integer. Henceforth, we assume that 5 divides exactly one of s and t. We consider
100
two cases. Suppose 5 divides t. Then b = 75 and we have ta + 75s = 2006 50. Subtract this from
75t +75s = 2006 75, we have (75 a)t = 2006 25. Since 5 divides t, 75 a divides 2 17 59 5.
Since 0 < a < 50, 25 < 75 a < 75. Hence we must have 75 a = 34 or 59. If a = 41, both t
ta
and a are odd, and 100
will not be an integer. If a = 16, we have t = 850. This leads to s = 1156.
Thus 867 students and 136 teachers responded to the survey, yielding a total of 1003, as required by
c = 50. Finally, since (100 a)t + (100 b)s = 100(s + t) (at + bs) = 200600 100300 = 100300,
the only other solution is b = 100 16 = 84 and a = 100 75 = 25, as indicated above since now 5
divides s instead of t. Solving s + t = 2006 and 84s + 25t = 100300, we have s = 850 and t = 1156.
Thus 714 students and 289 teachers responded to the survey, again yielding the desired total of
1003.

Alberta High School Mathematics Competition


Solutions and Comments to Part II, 2008.
Problem 1.
We have f (3n + 1) = 3an + b + a, f (3n) + 1 = 3an + b + 1 and 3f (n) + 1 = 3an + b + (2b + 1).
Hence a, 1 and 2b + 1 must be three consecutive integers in some order. Since one of them is 1,
they can only be (1, 0, 1), (0,1,2) or (1,2,3). Since 2b + 1 is odd, the second case is impossible. In
the first case, we must have 2b + 1 = 1 and a = 0, yielding f (n) = 1. In the third case, we must
have 2b + 1 = 3 and a = 2, yielding f (n) = 2n + 1.
Problem 2.
Let problem 1 be solved by students A, B and C. Since each pair of problems was solved by exactly
one student, every other problem was solved by exactly one of A, B and C. Suppose the number of
problems is at least eight. By the Pigeonhole Principle, at least three of the other seven problems
were solved by one of A, B and C, say A. Let these be problems 2, 3 and 4. Then apart from
A, no student solved more than one of problems 1, 2, 3 and 4. Since A did not solve all prolems,
there is one, say problem 5, which A did not solve. In order to have a common solver with each of
problems 1, 2, 3 and 4, problem 5 must be solved by at least four students, which is a contradiction.
Hence the number of problems is at most seven. The following scheme, which has all the desired
properties, shows that the number of problems can be exactly seven.
Students Problems Solved
A
1,2,3
B
1,4,5
C
1,6,7
D
2,4,6
E
2,5,7
F
3,4,7
G
3,5,6
Problem 3.
na
Let the total number of canides be n initially. After Autumn has taken 100
+ a candies, Brooke
na
b
will take (n 100 a) 100 + b candies. Equating these two expressions and simplifying, we obtain
ab
ab + 100b. This may be rewritten as (n + 100)(b a 100
) = 0. Since
na + 100a = nb nab
100
100a
n 6= 100, the second factor must be zero so that b = 100a . Note that a < 50 so that 100 a > 50.
Let p be any prime divisor of 100 a. Then p must divide 100a so that it divides either 100 or a.
It follows that p must divide 100, so that p can only be 2 or 5. This means that 100 a = 64 or
80. However, 36 does not divide 6400. Hence (a, b)=(20,25) is the only possibility. This does work
if we take n = 5k where 3k > 40. Autumn will take k + 20 candies, leaving behind 4k 20. Then
Brooke will take (k 5) + 25 = k + 20 candies.
Problem 4.
Let r and s be the two real roots of x2 + ax + b = 0. Then r + s = a and rs = b. That
(x2 2cx + d)2 + a(x2 2cx + d) + b = 0 has no real roots means that neither x2 2cx + d = r
nor x2 2cx + d = s has any real roots. It follows that (2c)2 < 4(d r) or c2 < d r. Similarly,
c2 < d s. Multiplication yields c4 < d2 d(r + s) + rs = d2 + ad + b.

Problem 5.
Since AB = AC and 6 CAB = 100 , 6 ABC = 6 BCA = 40 . Now 6 CAD = 6 CDA = 70 since
AC = DC, so that 6 BAD = 30 . Since ED is parallel to AC, 6 F DA = 70 and 6 BDF = 40 . Let
P be the point on CD such that 6 P AD = 6 F AD = 30 . Then 6 P AC = 40 so that P A = P C.
Now triangles P AD and F AD are congruent, so that AP = AF . Since 6 P AF = 60 , P AF is an
equilatral triangle. Hence 6 AP F = 60 so that 6 F P D = 20 . Moreover, P F = P A = P C. Hence
6 P CF = 6 P F C = 10 .
A
......
............
.. . . .
.... .. ... .....
..... .. .. .......
....
.... ... ....
....
....
. ...
.
.
.
.
....
.
..
..
....
.....
..
..
....
....
.
.
.
.
.
..
....
..
.
.
.
.
....
.
.
...
...
....
..
.
.
.
.
.
...
....
.
....
.
.
.
....
.
...
..
.
.
....
.
.
.
...
....
....
..
.
.
.
.
....
..
.
...
.
.
.....
.
.
.
.
...
...
.
....
.
.
.
.
.
.
...
.
....
... .................................................
.
.
.
.
.
....
..
.... ........... .....................
..
.
.
.
....
.
.
.
.
.
.
.
.
.
.
.
.
.
.
...................
....
..........
...
....
...
.
.
.
.
.
.
.
.
.
.
.
.
.
.
.
.
.
.
.
..........
....................
....
....
...
.
.
.
.
.
.
.
.
.
.
....
.
.
.
.
.
.
.
.
.
.
.
.
.
...................
....
...
. ...............
..
....
.
.
.
.
.
.
.
.
.
.
.
.
.
.
.
.
.
.
.........
...................
.... .
....
..
...
.
.
.
.
.
.
.
.
.
.
.
.
.
.
.
.
...
.
.
.
.
..... .
..........
.. .
.
...........................................................................................................................................................................................................................................................................................

Alberta High School Mathematics Competition


Solutions and Comments to Part II, 2009.
Problem 1.
Since (w + y) + (x + z) = 100, we have w + y = 50 + t and x + z = 50 t for some real number t.
Hence wx + xy + yz (w + y)(x + z) = (50 + t)(50 t) = 2500 t2 2500. This maximum value
may be attained for instance when w = x = 50 and y = z = 0.
Problem 2.
The number of integers between a2 and b2 inclusive is b2 a2 + 1. The number of squares between
a2 and b2 inclusive is b a + 1. From b2 a2 + 1 = 100(b a + 1), we have (b + a 100)(b a) = 99.
Since 99 has 6 positive divisors, there are 6 solutions, as shown in the chart below.
b + a 100
1
99
3
33
9
11

b a 2b 100
99
100
1
100
33
36
3
36
11
20
9
20

b
100
100
68
68
60
60

a
1
99
35
65
49
51

Here is a slightly different approach. Let d = b a. Then there are (a + d)2 a2 + 1 = 2ad + d2 + 1
integers under consideration, d + 1 of which are the squares of integers. It follows that we need
100(d + 1) = 2ad + d2 + 1, so that
a=

100 d 99
100(d + 1) d2 1
=
+ .
2d
2
2d

If d is even, the first term is an integer and the second is not. Hence d must be odd. Then the
first term is a fraction with denominator 2, so that the second term must also be a fraction with
denominator 2. This means that d must be a divisor of 99, that is, d is 1, 3, 9, 11, 33 or 99.
If d = 1, then a = 99
+ 99
= 99 and b = 99 + 1 = 100.
2
2
97
= 65 and b = 65 + 3 = 68.
If d = 3, then a = 2 + 99
6
91
99
If d = 9, then a = 2 + 18 = 51 and b = 51 + 9 = 60.
+ 99
= 49 and b = 49 + 11 = 60.
If d = 11, then a = 89
2
22
99
+
= 35 and b = 35 + 33 = 68.
If d = 33, then a = 67
2
66
1
99
If d = 99, then a = 2 + 198 = 1 and b = 1 + 99 = 100.
Problem 3.
There are 9 squares at the intersections of even-numbered rows and even-numbered columns. Any
2 2 block chosen by Betty must include one of these 9 squares. Hence Greta should play only on
these squares in her first four moves. This will ensure that Betty has at most five moves, and can
paint at most 20 squares brown. Hence Greta wins.

. ... .... .
. ... .... .
. ... .... .
. ... .... .
. . .. .

. ... .... .
. ... .... .
. ... .... .
. ... .... .
. . .. .

. ... ... ..
. ... ... ..
. ... ... ..
. ... ... ..
. . .. .

. ... .... .
. ... .... .
. ... .... .
. ... .... .
. . .. .

. ... .... .
. ... .... .
. ... .... .
. ... .... .
. . .. .

. ... ... ..
. ... ... ..
. ... ... ..
. ... ... ..
. . .. .

. ... .... .
. ... .... .
. ... .... .
. ... .... .
. . .. .

. ... .... .
. ... .... .
. ... .... .
. ... .... .
. . .. .

. ... ... ..
. ... ... ..
. ... ... ..
. ... ... ..
. . .. .

Problem 4.
Denote the circumcentre of by O and note that it lies within the circle with radius 23 . We have


1
BC = 4BD = 4(OB OD ) = 4 1
9
2

and

32
9

49
288
.
=
CA = 4CE = 4(OC OE ) = 4 1
121
121
2 +OC 2 BC 2
2 +OA2 CA2
23
= 79 and cos COA = OC 2OCOA
= 121
. Hence
By the Cosine
Law, cos BOC = OB

2OBOC
4 2
84 2
sin BOC = 9 and sin COA = 121 . It follows that
2

cos AOB = cos(6 BOC 6 COA)) = (cos BOC)(cos COA) + sin BOC sin COA =

833
.
1089

512
. Hence we have
By the Cosine Law again, AB 2 = OA2 + OB 2 2OA OB cos AOB = 1089
961
31
1
2
2
2
OF = OA AF = 1089 and OF = 33 . It follows that the radius of the third circle is 12 (1 31
) = 33
.
33

.....................................................
..................
..........
..
............
............. .....................
..........
....
.
.
....
.. ..................
.
.
.. ......
.
...
.
.
..
.....
..
.
....... ... .............
.
.
....
.
.
..
.
.
..
..........
....
..
.....
...
..........
..... ...
....
....
..
.
.... .....
.
.
.
.
.
.
.
.
....
.
.
.
.
.
.
.
..
..........
..
.
.
... .....
....
.
.
.
.
.
.
.
.
.
.
.
.
.
.
..........
..
...
.
...
.
.
... .....
.
.
.
.
.
.
.
.
...
.
.
.
...
......... ...
.
.
.......... ....
.
.
...
.
.
.
.
.
.
.
.
.
.
.
.
.
............ ..
...
..
...
.
..... ..........
.
.
.
.
.
.
.
.
.
.
.
...
.
.
..
.........................
.
... ..................
...
.
.
.
.
..........
...
.
....
...
.
.. ....
.
.
.
.
.
.
..........
..
.....
.
...
.. .....
.
.
.
.
.
.
.
.
.
.
..
..........
....
...
.
.. ....
.
.
.
.
.
.
.
.
.
.
.
.........
....
..
...
.
. .....
.
.
.
.
.
..
.
.
.
.
.
.........
.....
...
.
..
.
.
.. ....
.
.
.
.
.
.
.
.
...
....
..........
..
.
....
.
.
.
.
.
.........
.
.
...............................................................................................................................................................................................................................................................................................................
.
...............
.............
.....
.
... ...
.
.
.
.
.
.
.
.
.
.
.
.... ...................
.
.
.
.
.
.
.
.
.
.........
.... ......... ..
..
.
......
.
.
.
..........
.
.
.
.
.
.
.
.
.
.
.
.
.
.
.
...
.
.
.
.
.
.
.
..
.
.......
..........
...
....
.
..........
.....
..
..........
....
..........
..
..
..
..... ........
..........
.....
..
.........
...
...
... ....
....
.......... .......
..... ...................
...
.
.
...
.
.
.
.
.
.
.
.
.
.
.
.
.
....
... .. ..
............
..
.
..................
.
.
.
.
.
.
.
.
....
.
.
.
.
.
.... .. .. .
..........
....
.....
..
.
.
...
.
.
.
.
.
.
.
.
...
.
.
.
.
.
.
.
.... .. .. .
..........
...
......
.
..
.
.
...
.
.
.
.
.
.
.
...
.
.
.
.
.
.
.
.
.
.
.
.
.
.
.......... ... .. . .
...
...
...
.................... ...................
.
...
.
...
....................
..
...
.
.
....
..
..
..
...
.
..
.
.
..
..
...
..
....
.
.
..
..
.
.
...
.
...
.
.
..
.
.
.
..
..
.
.
.
.
..
...
..
.....
...
..
...
..
..
..
...
...
...
..
..
..
...
..
...
..
.
.
...
....
.
.
...
..
....
..
....
..
..
...
...
..
..
..
..
..
...
.....
....
..
..
.
.
.
.
.
.
.
..
...
..
..
..
....
...
..
..
...
..
...
...
...
...
..
..
...
...
..
...
.
..
.
.
.
...
.
..
..
...
....
...
..
..
...
...
...
..
..
...
...
..
...
..
....
....
...
.
.
..
...
.
.
.
.
...
.
....
...
....
....
...
...
....
....
...
...
...
....
....
....
...
.....
....
.....
...
....
.
.
.
.
..
.
..... .......
.
.
.... ........
.....
...
....
.
...... ........
....
.... ......
....... .....
..... .......
........ .....
..
......................
......... .......
.
.
.
.
.
.
.
.
.
....................
...................
.
.
.
.
.
..
......................
.
.
.
.
.
.
.
.
.
.
.........................................................
...
........
A
.......
............
........ .............

Problem 5.
We have (a + 1)a+1 = (a + 1)a (a + 1) = a(a + 1)a + (a + 1)a . Choose a = 33t for an arbitrary
3t1
positive integer t. Then a = (3t )3 and (a + 1)a = ((33t + 1)3 )3 are both cubes. If we take
3t1
3t1
k = 33t + 1, m = 3t (33t + 1)3
and n = (33t + 1)3 , then k k = m3 + n3 . Since t is an arbitrary
positive integer, the number of possible choices for k is infinite.
Essentially the same solution is given by Jarno Sun of Western Canada High School, who took
k = 27n3 + 1 for any positive integer n. Then
(27n3 + 1)27n

3 +1

= (27n3 + 1)27n

3 +1

(27n3 + 1)27n + (27n3 + 1)27n

= (27n3 + 1)27n (27n3 + 1 1) + (27n3 + 1)27n


3

= ((27n3 + 1)9n 3n)3 + ((27n3 + 1)9n )3 .


Yuri Delanghe of Harry Ainlay High School argued as follows. Let t be any integer congruent to
t2t 1
4 modulo 6. Then t 1 2t (mod 3). Hence t2t 1 is divisible by 3. Let m = n = 2 3 and
t
t
k = 2t . Then m3 + n3 = 2t2 = (2t )2 = k k .
When t = 4, k = 16, and this was also the initial example of Danny Shi of Sir Winston Churchill
High School. He showed inductively that for any solution k, 64k is also a solution, in that
(64k)64k = (464k k 21k )3 k k = (464k k 21k a)3 + (464k k 21k b)3 ,
where k k = a3 + b3 .

The Alberta High School Mathematics Competition


Solution to Part II, 2010.
1. We have 11x + 5y + 3(100 x y) = 1000 or 4x + y = 350. Since y 0, we get x 87.
Since x + y 100, we also have that 3x 250, so x 84. Thus the only solutions are
(x, y) = (84, 14), (85,10), (86,6) and (87,2).
2. For either (a) or (b), clearly the leading coefficient t of the quadratic must be positive.
(a) For the inequality to hold for all real x, the discriminant must be non-positive, that is,
0 (2t 1)2 4t(5t 1) = 1 16t2 = (1 4t)(1 + 4t).
Since t > 0, 1 + 4t > 0, so we need 1 4t 0. Thus t 14 .
(b) We now have the additional possibility that the two roots of the quadratic are real and
non-positive. This holds if and only if 0 < t 14 , 2t 1 0 and 5t 1 0. This is
equivalent to 15 t 14 . Combining with the answer to (a), we have t 15 .
3. First Solution:
Putting AB = BC = b and CD = c, we get AD = b + c. Let 6 BAD = . Since ABCD
is cyclic, 6 BCD = 180 . Applying the cosine law to triangles BAD and BCD, we have
BD2 = b2 + (b+ c)2 2b(b+ c) cos and BD2 = b2 + c2 2bc cos(180 ) = b2 + c2 +2bc cos .
Hence b2 + (b + c)2 2b(b + c) cos = b2 + c2 + 2bc cos , so that b2 + 2bc = (2b2 + 4bc) cos .
This yields cos = 12 , so that = 60 is the only possibility.
..... ....
..................... ...................................
........
.
.
.......
......
... ..............
....
.....
...
....
.
.
.. ... .....
.
.
.
.
....
.. .. ....
..
...
.
.
.
...
.
.
.
.
.. ... .....
..
..
.
...
.
.
.
.
.
..
.. .... .....
...
.
...
.
.
.
....
.. ..
...
.
...
.
.
.
.
.
.
....
.
..
..
.. ...
.
.
.
...
.
....
.
.
..
..
...
.... ...
... ......
..
... ..
...
..
... ......
.... ..
..
.
.. ....
.
......
..
...
......
.
...
...
..
...................
.
..
...
.............
...
....
..
.............
...
...
.......
..
............
.
.
.
.
.
.............
..
... ....
............. ....
..
.
.
. ..
.............
..
............. .... ......
..
.................
...
.
...
...
...
...
...
...
....
.....
....
.
.
.
.
.....
.......
......
...........
.......
......................................

Second Solution:
Let E be the point on AD such that DE = DC, so that AE = AD DE = BC = AB. Now
6 BDE = 6 BDC since they are subtended by the equal arcs BA and BC. It follows that
triangles BED and BCD are congruent, so that BE = BC = BA = AE, triangle BAE is
equilateral and 6 BAD = 60 .
4. First Solution:
The area of the punctured board is 22n 1. The base-2 representation of this number consists
of 2n 1s. Since the area of each rectangle in the partition is a power of 2, we must have at
least 2n rectangles. There exist such partitions with exactly 2n rectangles. Divide the board
in halves by a horizontal grid line. Set aside the one with the missing square and cover the
other with a rectangle of height 2n1 . Repeating the process with the strips set aside, we
obtain rectangles with decreasing heights 2n2 , 2n3 , . . . , 21 and 20 , a total of n rectangles.
We now divide the resulting 2n 1 board in halves by a vertical line. Set aside the one with
the missing square and cover the other with a rectangle of width 2n1 . Repeating the process
with the strips set aside, we obtain another n rectangles with decreasing widths, for a total
of 2n rectangles in the overall partition.

Second Solution:
Divide the board into four congruent quadrants. Set aside the one with the missing square.
Merge two of the other quadrants into one rectangle and keep the third quadrant as the second
rectangle. In reducing a 2n 2n board down to a 2n1 2n1 board, we use two rectangles. It
follows that we will use exactly 2n rectangles in the overall partition. We now prove that we
cannot get by with a smaller number. The area of a rectangle of the prescribed type is a power
of 2. The smallest has area 1, and the largest has area 22n1 . Thus there are 2n different sizes.
If we use one of each size, the total area of these 2n rectangles is 1 + 2 + + 22n1 = 22n 1,
exactly the size of the punctured chessboard. Consider any other collection of rectangles
whose areas are powers of 2 and whose total area is 22n1 1. Replace any pair of rectangles
of equal area by one with twice the area. Repeat until no further replacement is possible. The
resulting collection consists of rectangles of distinct areas which are powers of 2, and with
total area 22n1 1. It can only be our collection, and since mergers only reduce the number
of rectangles, 2n is indeed minimum.
5. (a) Note that f (M + m) f (m) is a sum of terms of the form ak ((M + m)k mk ) where ak is
the coefficient of the term xk in f (x). Since each term is divisible by M = (M + m) m,
so is f (M + m) f (m). Since M is divisible by f (m), f (M + m) f (m) is divisible by
f (m). It follows that f (M + m) is divisible by f (m).
(b) Since all the coefficients of f are non-negative and f is non-constant, it is strictly increasing. Let M = f (2)f (3) and n = M + 2. By (a), f (n) is divisible by f (2) and f (n + 1)
is divisible by f (3). Since f (n + 1) > f (n) > f (3) > f (2) > f (1) 1, both f (n) and
f (n + 1) are composite.

The Alberta High School Mathematics Competition


Solution to Part II, 2011.
Problem 1.

10, so the area of the circle


The diameter
of
the
circle,
being
the
diagonal
of
a
1

3
rectangle,
is

2
.
The
diagonal
of
the
square
is
4,
so
the
side
of
the
square is 42 = 8. Since
is ( 10/2)2 = 5
2
, we have 5
< 8. Thus the square has greater area than the circle.
< 3.2 = 16
5
2
Problem 2.
Eliminating z, we have x2 + y 2 = 2(t x y) so that (x + 1)2 + (y + 1)2 = 2(t + 1). In order to
have a unique solution for x and y, we must have 2(t + 1) = 0 or t = 1.
Problem 3.
Let M be the point on P Q such that 6 MAP = 6 BAP . Then
6

MAQ =

P AQ 6 MAP
1
(6 ABC 6 MAB)
=
2
1
6 MAC
=
2
= 6 CAQ.
6

Since 6 XP A = 6 MP A, triangles XAP and MAP are congruent by the ASA Postulate, so that
P X = P M. Similarly, we can prove that QY = QM, so that P X + QY = P M + QM = P Q.
A..
...........
....... ...
........... .. ..
.......... .. .
................ .... ...
.
.
.
... .. .. ... ...
..
..... ... ...
..
.... ..... ...
..
...
..... ... ...
..
..
... ..... .....
.
.
..
.
..
... ..... ....
.
..
.
.
.
.
..
... .... ....
...
.
.
.
.
...
.
.. ..... ....
..
.
.
.
.
...
.
.... ..... ....
.
.
.
..
.
.
. ...
..
...
.
.
..
.
.
.
.
.
.
.
.. ...
...
.
.
..
.
.
.
.
.
.
..
.
..
.
.
.
.
.
.
.
.
..
.
.
.
.
.
..
.....
..
.
.
.
.
..
.
.
.
.
.
.
.
..
..
.
.
.
.... ...
..
.
.
.
.
.
.
.
..
..
..
.
.
.
.
.
..
.
.
.
.
.
.
.
.
.
...
... .....
..
.
..
.
.
.
.
.
.
.......................................................................................................................................................................
........
..
.........
........
......... ...
........ ..
.....

B P M

C
Y

Problem 4.
Since f (n) takes on only positive integral values, it has a minimum value m. Let n be such that
f (n) = m. Then 2m f (n1)+f (n+1) 2f (n) = 2m, which implies that f (n1) = f (n+1) = m
also. It follows easily that f (n) = m for all integers n.

Problem 5.
We first show that the conditions of the problem can be satisfied. Construct a graph where the
teams are represented by vertices Ti , 0 i 6. In the diagram below, we partition the graph into
three subgraphs. Two teams play each other in the first sport if and only if the vertices representing
them are joined by an edge in the first subgraph, the second sport in the second subgraph and the
third sport in the third subgraph. None of the subgraphs contains a triangle.
T

... 0
....... ..............
........
........
.......
.......
.......
........
.
.
.
.
.
.
.
.....
... 1
6 .........
..
...
...
..
...
....
...
..
...
...
...
...
...
...
...
...
.
5 .....
...
.
.
..
...
.
.
.
...
.
.
.
...
...
...
....
...
...
...
...................................................

T
T

T2

T4

T3

.. 0
... ...
... ......
...
...
...
.
6 ............................................................................................... 1
...
..
..
... ....
.. ....
... .
... ....
... ..
......
....
.
..
.....
......
.. ....
.
.. ...
.
.. ......
... ...
.
.
...
..
..
..........
.
.
5 ........... ...
.......
...............
.........
............
.........
.
.
.
.
.
.
.
.. .........
..
....... .............. ...
..
..
..
..................
........ ...
... ..............
....
....

T
T

T2

T4

T3

T......0

.....
.. ..
.. ..
.. ....
........
.
1
.......
.
..........
...........
.. ....
... ........
. ......... ...
... ........ ....
......... ................. .....
...
.
.
.
...
.
.
.
.
.
.
.
... ........................... ....
............
..............
.. .
..... ...........
........ .......
........
.
....
........
.
.
.
.......
.
.
.
.
......
.....
..
.......................................................................................................................
5
...
......
....
.. ............ ....
... ...
.. ...
... ..
... ....
... ..
... ..
... ....
......
.........
..

T6
T

T2

T4

T3

The edges in the same subgraph have the same length, and those in different subgraphs have
different lengths. In geometric terms, a diverse triple is a scalene triangle. There is basically one
such triangle, namely T0 T1T3. Six others can be obtained from it by rotation, and seven more by
reflection. Thus we may have as many as 14 scalene triangles.
We now prove that there are at most 14 diverse triples. Construct a complete graph on 7 vertices
which represent the 7 teams. Paint an edge in the i-th colour if the teams represented by its
endpoints play each other in the i-th sport, 1 i 3. A triangle is diverse if all three sides are
of different colours, and non-diverse otherwise. Since there are no monochromatic triangles, a nondiverse triangle has two sides of the same colour. Call the vertex at the junction of the two sides
of the same colour its pivot. The number of pivots is equal to the number of non-diverse triangles.
There are six edges incident with each vertex. If at least 3 of them are of the same colour, then
this vertex is the pivot of at least 3 non-diverse triangles. If not, then exactly 2 edges are of each
colour, so that the vertex is the pivot of exactly 3 isosceles triangles. Hence each vertex is the pivot
of at least 3 non-diverse triangles. Since there are 7 vertices, this brings the total to at least 21, so
that the maximum number of diverse triangles or diverse triples is 14.

The Alberta High School Mathematics Competition


Solution to Part II, 2012.
Problem 1.
Let the dimensions of the lawn be a metres by b metres. The area of the first three rings is given
by ab (a 6)(b 6) = 6(a + b) 36. Similarly, the area of the next four rings is given by
(a 6)(b 6) (a 14)(b 14) = 8(a + b) 160. These two regions contain the same amount of
grass and so must be the same area. Thus 6(a + b) 36 = 8(a + b) 160. It follows that the only
possible value of the perimeter of the lawn is 2(a + b) = 124 metres.
Problem 2.
Since 6 DAB = 6 CBA and AB is parallel to DC, we have AD = BC. Since AB is parallel to DC,
6 BP C = 6 P CD. It follows that triangles BP C and P CD are similar. A similar argument shows
AP
A
)2 = BC
AD
= PP B
, as desired.
that triangles ADP and P CD are also similar. Hence ( PP D
C
BP
A......................................................................................P
B
....................................................

.. .
..
...
...... ......
...
..
.......
....
..
..
.......
...
..
..
.....
.
.
.
.
.
..
....
.
.
.
....
..
.....
.
.
..
.
.
.
....
..
...
.
.
..
.
..
.
.
.
...
....
.
.
.
.
...
.
.
.
... ....
....
.
.
.
.
...
.
.
.
... ...
...
.
.
.
.
...
.
.
... ...
..
.
......
.... ..
.. ........
..
..............................................................................................................................................................

D
C
Problem 3.
(a) For any positive integer n, n2 + 2 is special.
(b) We claim that for infinitely many positive integers n, n2 is not special. Suppose n2 = m2 + p
for some integer m and some prime number p. Then p = n2 m2 = (n m)(n + m). We
must have n m = 1 and p = n + m = 2n 1. If we let n = 3k + 2 for any positive integer
k, then 2n 1 = 6k + 3 is not a prime number. This justifies the claim.
Problem 4.
Extend AP and AQ to cut BC at M and N respectively.
Then ABM and ACN are isosceles,

so that BM = 2 and NC = 2 2. Hence MN


= 2 2 2. Now P Q is the segment joining the
MN
midpoints of AM and AN . Hence P Q = 2 = 2 1.
A
...
...............
. ..
.... ... .. .............
.... .. ..
.......
.... ... ....
.......
.
.......
....
..
.
.
.
.
.......
..
.
.......
....
..
..
....
.......
.
.
.
.
.
.
.......
...................................
...
.
.
......
.
.
.
.
.
.
.
............. ...................
..
.......
.
.
.
.
.
.
.
.......
.. ...................
.........
.
...
.
.
.
.
.
.
.
......
.
.
.
.
.
.
..............
..
.
...
......
.
.
.
.
.
.
.
.
.
.
.
.
.
.
.
.
.
.
.
.
.
.............. .............
..
.
.. ..............
.
.............. .......
.
.
.
.
..
..
............. ......
..................
.
.
.
.
.... ..
............................................................................................................................................................................................................................

Problem 5.
Suppose the real numbers x1, . . . , xn , 1 xi 4 for i = 1, 2, . . . , n, satisfy the two given inequalities.
Then (xi 1)(xi 4) 0 so that xi + x4i 5. Equality holds for xi = 1 or xi = 4. From these
inequalities and the given ones, we obtain
5n =

1
1
1
7n 8n
+
x1 + x2 + + xn + 4
+
+ +
3
3
x1 x2
xn

5n.

Hence xi = 1 or 4, i = 1, 2, . . . , n. Suppose x1 = x2 = = xk = 1 and xk+1 = xk+2 = = xn = 4


. Hence 5n = 9k so that 9 divides
for some index k. Then x1 + x2 + + xn = k + 4(n k) = 7n
3
n. It follows that the smallest value of n is 9, with the numbers 1, 1, 1, 1, 1, 4, 4, 4 and 4.

The Alberta High School Mathematics Competition


Solution to Part II, 2013.
Problem 1.
The given equation may be rewritten as ab +

36
ab

+ 12 = 25. Therefore

(ab)2 13ab + 36 = (ab 4)(ab 9) = 0.


Hence ab = 4 or ab = 9. Note that a and b are positive integers with a b. If ab = 4, we have
(a, b) = (1, 4) or (2,2). If ab = 9, we have (a, b) = (1, 9) or (3,3). It is easy to verify that all four
are indeed solutions.
Problem 2.
(a) Candys perfect set may be {1, 2, 3, 5, 7, 11, 13, 17, 19}. We claim that this number is the
highest possible. Now a maximal perfect set must contain the element 1, as otherwise we can
add 1 and obtain a larger perfect set. Also, a maximal perfect set cannot contain an element
which is divisible by two distinct primes, as otherwise we can replace that element by the two
primes and obtain a larger perfect set. Hence each element pther than 1 is a positive power
of a prime. Moreover, distinct elements are powers of distinct primes. Since there are only 8
primes less than 20, namely, 2, 3, 5, 7, 11, 13, 17 and 19, we claim is justified.
(b) Every maximal perfect set Candy can build must have the form
S = {1, 2i2 , 3i3 , 5i5 , 7i7 , 11i11 , 13i13 , 17i17 , 19i19 },
where each exponent is a positive integer. Since 52 > 20, the exponent for all primes greater
than or equal to 5 must be 1. Since 24 20 25 and 32 20 33 , the exponent for 2 must
be 1, 2, 3 or 4, and the exponent for 3 must be 1 or 2. This yields eight different maximal
perfect sets.
Problem 3.
 
Let n 2 be the number of rays drawn by Randy. Then there are n2 = n(n1)
pairs of rays. Each
2

pair determines two angles adding up to 360 . Hence the total number of angles between 2 of the n
rays is exactly n(n 1). The measure of such an angle is clearly less than 360 . Since it is supposed
to be an integral multiple of 10 , there are at most 35 values for the measures of these angles. Since
they are distinct, n(n 1) 35. Now 6 5 = 30 < 35 < 42 = 7 6. Hence n 6. It is possible
for Randy to draw 6 rays, determining 30 distinct angles. In the diagram below, 6 U AV = 60 ,
6 V AW = 40 , 6 W AX = 10 , 6 XAY = 20 , 6 Y AA = 140 and 6 ZAU = 90 .
..............................
X..........................W
......... V
......
.. ........
Y......................... ........ .........
....
...
..
..
...
....
.. ..
..
...
.. ...
...
...
..
..
.
...
...
...
... .... ....
...
...
..
... .. ...
..
..
.
..
.
.
.
... .. ..
..
.
.... ...... ....
.
..
... .... ..
...
..
... ..... ..
..
.
.
.
........ ..
.......................................................................
...
...
...
..
...
...
...
...
..
..
...
...
..
.
.
.
..
...
.
..
..
...
...
..
...
..
...
...
...
...
.
.
.
....
...
...
....
...
....
.....
...
....
....
....
...
.......
......
.
.
.
.........
.
.
.
.
.
...........................................

We now verify that the 30 angles between two rays are distinct. We have 6 W AY = 30 , 6 V AX = 50 ,
6 V AY = 70 , 6 U AW = 100 , 6 U AX = 110 , 6 U AY = 130 , 6 ZAV = 150 , 6 XAZ = 160 as well
as 6 W AZ = 170 . These are 9 different angles distinct from the 6 between adjacent rays. All have
measures less than 180 . Corresponding to these 15 angles, we have 15 other angles greater than
180 , yielding a total of 30 distinct angles.
Problem 4.
Let L be the point of intersection of EC and DB. Let M be the point on the extension of AB
such that M C is parallel to AE. Then ABLE and AMCE are parallelograms. Note that triangles
DLC and EAB are similar, as are triangles AMC and ELD. It follows that
EC
EL + LC
LC
DL
DL
AB
=
=1+
=1+
=1+
=1+
.
AB
AB
AB
EA
CM
EC

. Then x = 1 + x1 so that x2 x 1 = 0. Hence x = 1+2 5 . Similarly, we have


Let x = EC
AB

DB
AC
AD
EB
1+ 5
=
=
=
=
, so that EC + DB + AC + AD + EB = 5(1 + 5).
AE
ED
BC
DC
2
A

.......
.......... ....
.... .. .. ....
.... .... .... .......
....
..
.....
.
.
.
.
.
....
..
.
..
....
..
..
.....
....
..
..
....
....
..
..
....
.
.
.
....
.
.
.
.
.
..
....
....
.
.
.
.
.
.
....
..
.
..
.
.
.
....
.
.
.
.
..
.
..
...
.
.
.
.
.
.
.............................................................................................................................................................
.
.
.
..
... ..
.... ..........
..
.
.
.
.
..
.
..... ...
.. .....
..
.
.
.
.
.
.
.
.
.
.
..
..
....
.
.
....
..
..
....
..
....
..
....
..
..
....
....
..
.
..
.
.
........
.
.
.
.
.... ..
..
.. .....
.
...
.... ..
..
.. ...
..
....
.....
..
.
.
.
.
.
.
.
...
..
... ..
..
.. ..
....
..
.... ....
.. ........
..
...
..
....
..
....
..
..
....
..
..
.... ........
..
...
....
..
..
........
...
.
.
.
.
.
....
. ..
..
..
.
...
..
..... .......
..
..
..
...
..
....
....
..
.... ... ....
....
..
....
.
... .. ..
....
..... .... ....
.... .. .. .......
.
.
.
.
.... .. .
... .. ... .....
... .. ..
............
.................................................................................................

C
D
Remark:
The regular pentagon is used in the illustrative diagram. Many studentsmay get the correct answer
by treating only this special case, essentially proving that cos 36 = 1+4 5 .
Problem 5.
The conditions are:
r2 + br + c
s2 + bs + c
t2 + bt + c
r+t

=
=
=
=

1,
b,
c,
2s.

(1)
(2)
(3)
(4)

From (3), t(t + b) = 0 so that either t = 0 or t = b. We consider these two cases separately.
Case 1: t = 0.
From (4), we have r = 2s. Substituting into (1), we have 4s2 + 2bs + c = 1. Subtracting (2) from
this, we have 3s2 + bs = 1 b which may be rewritten as (s + 1)(3s 3 + b) = 2. Hence 2 is
divisible by s + 1, so that s = 3, 2, 0 or 1. However, since s > t = 0, we may only have s = 1.
It follows that b = 1. Hence f (x) = x2 x 1, with r = 2, s = 1 and t = 0.
Case 2: t = b.
From (4), we have r = 2s + b. Substitoting into (1), we have 4s2 + 6sb + 2b2 + c = 1. Subtracting (2)
from this, we have 3s2 + 5sb + 2b2 = 1 b which may be rewritten as (3s + 2b + 3)(s + b 1) = 2.
Hence 2 is divisible by s + b 1. From r > s > t = b, we have s + b > 0. Hence s + b 1 > 1 so
that s + b 1 = 1 or 2. If s + b 1 = 1, we have 3s + 2b + 3 = 2 so that s = 9 and b = 11. Hence
f (x) = x2 + 11x + 30 with r = 7, s = 9 and t = 11. If s + b 1 = 2, we have 3s + 2b + 3 = 1
so that s = 10 and b = 13. Hence f (x) = x2 + 13x + 43, with r = 7, s = 10 and t = 13.

The Alberta High School Mathematics Competition


Solution to Part II, 2014
Problem 1.
ABCD is a square. The circle with centre C and radius CB intersects the circle with diameter AB
at E 6= B. If AB = 2, determine AE.
Solution:

Let O be the midpoint of AB. Then OB = 1 and OC = 12 + 22 = 5. The line OC of centres of


the two circles is perpendicular to the common chord BE. AE is also perpendicular to BE since E
lies on the circle with diameter AB. Now 6 ABE = 90 6 COB = 6 OCB. Hence triangles ABE
= 2 5.
= 21
and OCB are similar, so that AE = ABOB
OC
5
5
D

.............................................................................................................................................................
...
.
.. ...
.. ..
.....
.. ..
........
... ..
... ....
.......
.
......
...
..
... ...
..
...
..
.... ....
....
..
... ..
..
.
..
... ...
.
...
..
.... ....
.
...
..
.
...
...
..
..
...
...
...
.
..
...
.
..
...
.
...
...
.
.
...
....................................... ..
...
.
.
.
.
.
.
.
.
.
.
.
....
...
.
..........
...
......
.
.
.
.
.
.
...
.
...
.
.
.
......
... .....
.
.
.
.
.
...
.
...
.
....
.
..................
.
.
.
.
.
...
...
.
.
....
.
...... ..................
.
.
.
...
...
.
.
....
.
.... ........
. ...
.
.
.
.
.
...
...
.
..
....
.
...
..
.
.... ..............
... .... ....
.
.
.
.. ...
.......
....
.
... ... ...
.
.
.....
........ ..
.. ...
.......
.....
.. ..
... .. ...
. ........
.....
.
.. ... ..
.
.. ...
.......
...... ...
.. ...
.......
....... ...
....... ..
....
........
..... ...
.
........
....
.................
.... ..
.
...
.
........
..........
..
.
.
.
.
.
.......
.
.
.... ...
.......
.
................................................................................................................................................................................

Problem 2.
A family consists of two parents of the same age, and a number of children all of different ages.
The average age of the children is 15, and the average age of the whole family is 21. When each
child was born, the parents were at least 25 and at most 35 years old. All ages are given in whole
numbers of years. Find all possible values of the number of children in this family.
Solution:
Let the age of the parents be p and let the number of children be n. Then the total age of the
family is 15n + 2p = 21(n + 2), which simplifies to p = 3n + 21. Since the children are of different
ages and their average age is 15, the age of the eldest one is at least 15 + n1
= n+29
. It follows
2
2
n+79
+
25
=
.
This
simplifies
to
5n

37
which
implies
that
n 8.
that 3n + 21 = p n+29
2
2
n1
31n
On the other hand, the age of the youngest child is at most 15 2 = 2 . It follows that
+ 35 = 101n
. This simplifies to 7n 59 which implies that n 8. It follows
3n + 21 = p 31n
2
2
that the only possible value is n = 8. This may be realized if the children are of ages 11, 12, 13, 14,
16, 17, 18 and 19, and both parents are of age 45.
Problem 3.
Two cars 100 metres apart are travelling in the same direction along a highway at the speed limit
of 60 kph. At one point on the highway, the speed limit increases to 80 kph. Then a little later,
it increases to 100 kph. Still later, it increases to 120 kph. Whenever a car passes a point where
the speed limit increases, it instantaneously increases its speed to the new speed limit. When both
cars are travelling at 120 kph, how far apart are they?

Solution:
Let the first car be at a point B while the second car is at a point A, both in the 60 kph zone. Then
AB = 100 metres. Let the first car be at a point D while the second car is at a point C, both in
the 120 kph zone. Now the amount of time the second car takes to go from A to C is the same as
the amount of time the first car takes to go from B to D. Both cars take the same amount of time
going from B to C. Hence the amount of time the second car takes to go from A to B at 60 kph
is the same as the amount of time the first car takes to go from C to D at 120 kph. It follows that
CD = 2AB = 200 metres.
Problem 4.
Let p(x)be a polynomial with integer coefficients such that p(1) = 5 and p(1) = 11.
(a) Give an example of p(x) which has an integral root.
(b) Prove that if p(0) = 8, then p(x) does not have an integrral root.
Solution:
(a) We are given two pieces of information. So we seek a polynomial with two undetermined
coefficients. The first attempt is p(x) = ax+b. Then 5 = p(1) = a+b and 11 = p(1) = a+b.
Hence a = 3 and b = 8, but the only root of 3x + 8 = 0 is x = 83 , which is not integral.
However, it is easy to modify our polynomial to p(x) = 8x2 3x. We have p(1) = 5 and
p(1) = 11, but this time, we have an integral root x = 0 in addition to x = 38 .
(b) Suppose p(x) has an integral root x = r. Then r 1 divides p(r) p(1) = 5, so that r is
one of 4, 0, 2 or 6. Also, r + 1 = r (1) divides p(r) p(1) = 11, so that r is one
of 12, 2, 0 and 10. The only common value between the two lists is r = 0, but p(0) = 8.
This is a contradiction.
Problem 5.
On a 2 n board, you start from the square at the bottom left corner. You are allowed to move
from square to adjacent square, with no diagonal moves, and each square must be visited at most
once. Moreover, two squares visited on the path may not share a common edge unless you move
directly from one of them to the other. We consider two types of paths, those ending on the square
at the top right corner and those ending on the square at the bottom right corner. The diagram
below shows that there are 4 paths of each type when n = 4. Prove that the numbers of these two
types of paths are the same for n = 2014.

Solution:
The path of the marker is uniquely determined by its vertical moves. The only condition is that
no two vertical moves can be made in adjacent columns. Whether the path ends in the upper or
lower right corner is determined by the parity of the number of vertical moves. Let the columns
be represented by elements in the set {1, 2, . . . , n}. Consider all subsets which do not contain two
consecutive numbers. Let an be the number of such subsets of even size, and bn be the number of
such subsets of odd size. Then a0 = a1 = a2 = 1 because of the empty subset, b0 = 0, b1 = 1 and
b2 = 2.

For n 3, classify the subsets of {1, 2, . . . , n} into two types, those containing n 1 and those not
containing n 1. A subset of the first type cannot contain either n 2 or n. Hence the number of
such subsets of even size is bn3 and the number of such subsets of odd size is an3 . The subsets
of the second type may be divided into pairs such that in each pair, the two subsets are identical
except that one contains n and the other does not. Hence the number of such subsets of even size
is equal to the number of such subsets of odd size. It follows that an bn = bn3 an3 . Hence
a3k b3k = (1)k (a0 b0 ) = (1)k ,
a3k+1 b3k+1 = (1)k (a1 b1 ) = 0,
a3k+2 b3k+2 = (1)k (a2 b2 ) = (1)k+1 .
In particular, a2014 = b2014.

The Alberta High School Mathematics Competition


Solution to Part II, 2015.
Problem 1.
Find the number of isosceles triangles of perimeter 2015 such that all three sides are odd integers.
Solution:
Let m, m and n be the lengths of the sides of the triangle. Then 2m + n = 2015. We should have
2m > n, hence n 1007. Thus (n, 2m) = (1, 2014), (3, 2012), (5, 2010), , (1005, 1010), (1007, 1008),
or taking into account that m is odd, (n, m) = (1, 1007), (5, 1005), , (1005, 505). The number of
pairs is (1007 505)/2 + 1 = 252 and hence there are 252 triangles.
Problem 2.
Find all pairs (m, n) of positive integers such that m3 n3 = 5mn + 43.
Solution:
Since m > n, k = m n is a positive integer. Then (k + n)3 n3 5(k + n)n = 43 or
(3k 5)n2 + k(3k 5)n + k 3 = 43

(1)

We cannot have k = 1 because then (1) simplifies to n2 +n+21 = 0 which has no real solutions.Thus
we assume k 2, which means that 3k 5 > 0. From (1), we get k 3 < 43, and so we have k 3. If
k = 2, then (1) simplifies to 0 = n2 + 2n 35 = (n 5)(n + 7), with the positive integral solution
n = 5. If k = 3, from (1) we get 0 = n2 + 3n 4 = (n 1)(n + 4), with the positive integral solution
n = 1. Hence there are two such pairs, namely, (m, n) = (7, 5) and (4,1).
Problem 3.
Let f : [0, 4] [0, ) be such that f (4) = 2 and f (x + y) f (x) + f (y) for any real numbers x
and y in the closed interval [0,4] such that x + y 4.
(a) Suppose that 0 x y 4. Show that f (y) f (x).
(b) Show that f (x) x for any x in [0,4].
Solution:
(a) If 0 x y 4 then f (y) = f (x + (y x)) f (x) + f (y x) f (x).
(b) If x is in [2,4], then f (x) f (4) = 2 x. On the other hand, if x is in (0,2), then
x2 = x2 > 1. Hence there is a positive integer n such that x2 < n < x4 or 2 < nx < 4, so that
f (nx) nx. Also, f (nx) = f ((n1)x+x)) f ((n1)x)+f (x) f ((n2)x)+2f (x) nf (x)
and hence f (nx) nf (x). Consequently nf (x) f (nx) nx, that is, f (x) x for any x in (0,2).
Also, from f (0) + f (0) f (0) we obtain f (0) 0, and since f (0) 0, we get f (0) = 0. Therefore
f (x) x for any x in [0,4].
4
x

Problem 4.
E and F are points on the sides CA and AB, respectively, of an equilateral triangle ABC such that
EF is parallel to BC. G is the intersection point of medians in triangle AEF and M a point on
the segment BE. Prove that 6 M GC = 60 if and only if M is the midpoint of BE.
Solution:
Let H be on the extension of EF such that BCEH is a parallelogram. Then 6 BHE = 6 BCE = 60
and 6 BF H = 6 AF E = 60 , so F BH is also an equilateral triangle, and in particular HF = F B =
EC. Also 6 HF G = 180 6 GF E = 180 30 = 150 , and similarly 6 CEG = 150 , thus
6 HF G = 6 CEG. Since 6 GF E = 30 = 6 GEF , we know F G = GE. Hence triangles HF G and
CEG are congruent (by SAS) so that GH = GC and 6 HGF = 6 CGE. It follows that 6 HGC =
6 HGF + 6 F GC = 6 CGE + 6 F GC = 6 EGF = 180 6 GF E 6 GEF = 180 30 30 = 120 .
Now M is the midpoint of the diagonal BE of the parallelogram if and only if it is the midpoint of
the diagonal HC. Since triangle HGC is isosceles, this is equivalent to GM being the bisector of
6 HGC. In other words, 6 M GC = 60 .
A
..
......
. .
... .....
...
...
...
...
...
..
.
.
...
...
...
.
.
.
...
.
..
...
.
.
...
...
.
...
.
.
...
.
...
...
.
...
.
.
.
.
.
.
.
.
.
.
.
.
.
.
.
.
.
...
.. ................................. ..................
.
...
.
............. ........ .. ...............
.
...
.
.
.
.
.
.
.
.
..... ........
.
........... ..........
.
.
.
.
.
.
.
.
.
.
.
.
.
.
.
.
.
.
..... ....... ....
. ........
.
.......
.
.
.
.
.
.
.
.
.
.
.
.
.
.
.
.
.
.
.
.
.
.
..... ............
....
.......
.
.
.
.
.
.
.
.
.
.
.
.
.
.
.
.
.
.
.
.
.
.
.
.
.
...........................................................................................................................................................................................
.
.
.
.
.
... ...............
.......
...
.............. ....
...
...
.
...................
.........
...
..
..... .....
..........
..
..........
.. ..........................
..... ...
...
..................................
..... ..
...
...
.
.
.
.
.
.
.
.
.
.
.
.
.
.
.
...
..............
..... ....
......
..............
..... ...
...
...
..........
..............
..... ..
... .....
..........
..............
.. .
... ... ...................
.
.
.
.
.
........ ...........
................
..................................................................................................................................................................

@
R
@

Problem 5.
Karys is helping her father move basketballs from his car to the gymnasium. She carries either 3
or 4 basketballs each trip, while her father carries 6 or 7 basketballs each trip. Altogether Karys
makes 15 more trips and carries 15 fewer basketballs than her father.
(a) Determine the minimum number of basketballs that Karys carries.
(b) Determine the maximum number of basketballs that Karys carries.
Solution:
Suppose that Karys carries n basketballs in total. Then she will make at least n4 and at most n3
trips. Her father carries n + 15 basketballs in total, so he makes at least n+15
and at most n+15
7
6
trips. Thus the difference between the number of trips Karys makes and the number of trips her
father makes is at least n4 n+15
and at most n3 n+15
. Thus we get
6
7
n n + 15

15
4
6

and

n n + 15

15,
3
7

and

4n 45 315.

which simplify respectively to


2n 60 360
Thus 90 n 210.

(a) It is possible for Karys to carry 90 basketballs in total and for her father to carry 90+15=105
basketballs, because Karys can carry 3 basketballs at a time for 30 trips, while her father
carries 7 basketballs at a time for 15 trips, so Karys indeed makes 30 15 = 15 more trips
than her father.
(b) It is not possible for Karys to carry 210 basketballs in total. To do this she would need at least
210
= 52.5 trips, so at least 53 trips. Her father would take at most 225
= 37.5 trips, so at most
4
6
37 trips. So the difference in the number of trips is at least 53 37 = 16, not 15. Similarly,
209 basketballs carried by Karys and 224 carried by her father is not possible either, because
we get d 209
e = 53 and b 224
c = 37. However, 208 for Karys and 223 for her father is possible,
4
6
208
c = 37, and 52 37 = 15. Karys carries 4 basketballs at a
because now d 4 e = 52 and b 223
6
time for 52 trips for a total of 208 basketballs, while her father carries 6 basketballs at a time
for 36 trips and 7 basketballs at a time for 1 trip, for a total of 36 6 + 7 = 223 basketballs in
37 trips.
Remark:
Similarly we can check that Karys could carry 207 basketballs, but it turns out that 206
e = 52
basketballs carried by Karys (and 221 by her father) is again not possible, because d 206
4
221
and b 6 c = 36, and 52 36 = 16.

Potrebbero piacerti anche